Sei sulla pagina 1di 151

aAOTAM

('I ~

'\ ' \ r 'vV)

\~' '0J.'~~ v, l~VIi.J..

\ VJ" ( , "

\ ,/'r- 1\ ~~I-

• ,,\ V •

\ ' ,'/ ,~\I . \ ',y..\

'J /', I ,.\ V"\' , (') \ \ ,

, 1\ v '" ,

/// \\; \' \~\ , BQ GIAO Dl,JC vA BAo T.~O

/' TRUONG B~ HOC SIT P~M VINH

LE rn6NG NHAT

, A.... ?,

HEN LUYEN NANG Life GIAI rOAN

• a·

,:J A

CHO HOC SINH PHD THONG TRUNG HOC

• a

A A .... ,,?_

THONG QUA VIEe PHAN TICHVA SUA CHUA

a

, ""'? ? ,

CAe SAl LAM CUA HOC SINH KHI .GIAI TOAN



Chuyin ngimh : PHUONG pHA.P GIANG Df\. Y ToAN Mil so : 5.07.02 .

I ~

A, , AI' _ '" ,

LU~~N AN PHO TIEN SI KHOA HQC SU PH4i\'I - TAM L Y

N guiTi flirting di" khoa hQc:

vn DUONGTIIVY

Pho Gido SU, Pho ciln st IOwa hoc giao due

Phd Giao SU, PM tien si Khoa hoc gido due l

LOr CANi DOAN

Toi xin cam doan day la cong trinh nghien cuu cua rieng toi, Cac so lieu, ket qua neu trang luan an la trung thL;1C va chua tung duoc ai cong bo trong bat ki cong trinh nao khac ..

Lor CAlvI ON

Luan an duoc hoan thanh duoi su huang d~n cua PGS.PTS Vu Duong Thuy va PGS.PTS Dao Tam. Trong thoi gian hoan thanh luan an, rae gia con duoc su giup do cua Khoa Toan Truong Dai hoc Su pharn Vinh, Phong Toan Vien Khoa hoc Giao due, B¢ man Phuong phap day hoc toan-tin Truong Dai hoc Su pham - Dai hoc Qucc gia Ha Noi, Khoa Dao tao sau dai hoc Truong Dai hoc Su pharn Vinh, Phong Dao tao sau dai hoc Truong Dai hoc Khoa hoc tu nhien - Dai hoc Quoc gia Ha NQi, Vu Dao tao sau dai hoc B¢ Giao due va dao tao, Ban Toan Nha xuat ban Giao due. Dac biet tac gia nhan dUQC nhieu g6p y bo ich cua PGS. PTS Tam Thue Trinh. GS.TS Nguyen ss Kim, PGS. PTS Tran Kieu, PGS.PTS Ngo Huu Dung, PGS.PTS Nguyen Quy Dy, PTS Dao Thai Lai, PTS D6 Manh Hung, PTS Nguyen Vi~t HaL PTS Bui Gia Quang, PTS Nguyen Hiiu Chau, ...

Tac gia nhan duoc su giup do tan tinh ella eng Nguyen Kim Hoan (Giarn doc S6 Giao due va dao tao Ha Noi), eng Pharn Quy Hung (Ph6 Giarn doc S6 Giao due va dao tao Nghe An), eng Pharn Dinh Dau, (Hieu truong truong PTTH Chu Van An, Ha NQi), ong Nguyen Xuan Khang (Hieu truong truong PTTH dan l?p Marie-Curie, Ha NQi), ong Vt1 Due Thu (Hieu truong truong PTTH Le Hong Phong, Nam Ha) va nit nhieu GV Toan cac truong PTTH 6 : TP Ha N¢i, TP ne Chi Minh, Ha Thy, Nghe An. Ha Bac, Thai Blnh, Ninh Binh, Long An, Thanh H6a.

Trong thoi gian hoan thanh luan an, tac gia rat nho toi nhilng bai hoc d~lU tien v~ Phuong phap giang day to an cua thay giao Nguyen Van Bang (Dai hoc 5U pham Hue}

Cac thay giao Nguyen Thai Hoe (DHSP Vinh), Le Quang Phan (Vien KHGD) da danh cho rae gia nhieu loi khuyen be) ich.

Gia dinh va ban be than htiu luon la nhiing nguon dong vien rat hieu q ua de rae gia eo them nghi luc, tinh th~n hoan thanh luan an.

Tac gia luan an

Truce khi trinh bay noi dung chinh cua luan an, xin chan thanh earn On t,lt ca rnoi tam long da uu ai danh eho tac gia.

HS:

GV:

PTTH :

')

SGK:

CAC KI HIEU VIET T,i. T

HQc sinh Giao vien

Ph6 thong trung hoc Loi giai sai l§.m Phan tich sai lam Sach giao khoa

2

rvll)C Ll)C

Trang

ivle dau 3

Clurang 1 : ~GHIEN ctiu c.ic SAl LA.Yl PHO B[E~ CUA

HOC SINH PHO THONG mUNG HOC KHI GIAI TOAN 8

1.1. lll:h hinh thuc te qua cac dieu tra va quan sat 8

1.2. Mot so sai lam pho bien cua hoc sinh pho thong

trung hoc khi giai toan Dai so - Giai tich 14

1.3. Phan tich cac nguyen nhan dan toi sai lam

cua hoc sinh pho thong trung hoc khi giai to an 63

Chtrong 2 : CAC BI:t;:N PHAP REN LUYtN ~At'iG Ll,JC GIA! ToAN

CHO HOC SINH PHO THONG mUNG HOC THONG QUA

PHAN TiCH vA SUA CHUA SAl LAM 78

2.1. Co 56 Iv luan 78

2.2. Ba phuong cham chi dao su dung cac bien phap

su pharn nharn han che va sua chua sai lam cua

hoc sinh khi ziai roan 83

. -

. 2.3. Bon bien phap su pharn chu yeu nharn han che

va sua chua sai lam cho hoc sinh 87

2A. Cac yeu cau d6i voi hoc sinh va giao vien 114

Clurirng 3 : THl,fC NGHII;:M SU PH,A.M 127

3.1. Muc dich thuc nghiern 127

3.2. NQi dung thuc nghiern 127

3.3. T6 chuc thuc nghiern 127

,3"+. Ket qua thuc nghiern 130

3.5. Ket luan thuc nghiern 132

3.6. De nghi ve mot hieu biet quan trong cua

sinh vien su pharn toan 132

Ket luan 134

Cac cong trinh da cong bo lien quan toi luan an 135

Tai lieu tham khao 136

Phu luc : Kinh nghiern thuc ti~n cua rae gia 145

3

Nra DAU

1. LI DO CHON DE TAL

Hien phap nuoc CHXHCN Viet Nam nam 1992 da ghi 6 dieu 35 :

"Giao due - dao tao la quae sach hang dau". Bao cao chinh tri cua BCHTW Dang khoa VII tai Dai hoi dai bieu roan qucc l~n thu VIII cua Dang lai khang dinh : "Giao due va dao tao la quoc sach hang dau nharn nang cao dan tri, dao tao nhan luc, boi duong nhan tai".

Tam quan trong cua giao due va dao tao trong su nghiep cua dan toe dat len vai doi ngu nhung nguoi lam cong tac giao due nhieu trach nhiern nang ne,

"Trang cac men khoa hoc va ki thuat, toan hoc giu mot vi tri noi bat. No con la mon the thao cua tri tue, giup chung ta nhieu trong viec fen luyen phuong phap suy nghi, phuong phap suv luan, phuong phap hoc tap, phuong phap giai quyet cac van d~, gitip chung ta ren luyen tri thong minh sang tao.

~16 con giup chiing ta ren Iuyen nhieu due tinh qui ball khac nhu can ell va nhan nai, tu luc canh sinh. y chi vuot kho, yeu thich ehinh xac, ham chuong chan li, Oil cac ban phuc vu nganh nao, trong cong tac nao thi cac kien thuc va phuong phap toan hoc cung nIt can cho

~" b "["8 1]

cac ant ze, tr. ..

Cac nha giao day toan chinh la cac huan luyen vien trong men the thao tri tue nay. Cong viec day toan cua chung ta nharn ren luyen cho HS tu duy toan .hoc cung nhung pharn chat cua con nguoi lao dong rnoi dS cac em viing vang tro thanh nhung chu nhan tuong lai cua Oat nuoc,

o truong pho thong day hoc roan In day hoar dong roan hoc. Doi voi HS co the xern giai loan fa hinh lh;i-c chu yeu cua hoat dqng loan hoc.

Cac bai roan 6 truong pho thongla mot phuong tien rat co hieu qua va khong the thay the dUQC trong viec giup HS narn vung tri thuc, phdt trien tu duy, hinh thanh ki nang, ki xao ling dung roan hQC vao ~uJc song. Day hoc giai toan mang trong minh cac chuc nang: giao

,

duong, gino due, 'phat trien va kiern tra. Vi V?y heat dong giai roan la

dieu kien de thuc hien tot. cac muc dich day hoc toano Do do, t6 chuc co hie~ qua viec day hoc giai roan co vai tro quyet dinh d6i voi chat luong day hoc roan [53, tr. 201].

Khao sat thuc tien day hoc roan 6 nuoc ta trong nhieu narn qua co the thay ding chat luong day hoc roan 6 truong pho thong con chua tot,- the hien 6 nang luc giai roan cua HS con. han che do HS con vi pharn nhieu sai lam v~ kien thuc.:phudng phap to an hoc. Trang do nhieu GV con co it kinh nghiern trong cac viec : phat hien sai lam cua

, .

HS' khi giai roan, tim ra nhung nguyen nhan cua nhung sai lam do va

nhiing bien phap han che, sua chua chung, Trang cac an pharn nghien cuu khoa hoc giao due d nuoc tao cho toi nay con thieu vang nhiing

-_

cong rrinh nghien cuu co h~ thong ve linh Vl:!C nay. Tu do co nhu cau

nhan thuc ve sai lam. tim ra nhung nguyen nhanva nhung bien phap

-

han che, sua chua kip thoi cac sai lftm nay, nharn ren luyen nang luc giai

roan cho HS d6ng thoi nang eao hieu qua day hoc roan trong cac truong pho thong, dac biet 6 truong PITH de dat muc tieu dao tao chu dong hen, t6t hon. Chung toichon doi wong la HS truong PTTH vi b~c hyc nn co nhi~m V\l hoan chinh giao d\lc ph6 thong, ch1J~n bi cho HS ra cuoc song va mot bO ph;fln hQc len b~c Cao dAng, Trung h9c chuyen" nghi~p, D~i h9c.

5

/'

Tren the' gioi, nhieu nha khoa hoc n6i tieng da phat bie nhit!u y

kien bo ich ve v~n de nay. Chang h:n~A.Kome~sky da _ :i.ng _dinh :

"Bat ki mot sai lam nao cung co the l~ cho HS hoc em di neu nhu GY khorig chu y ngay toi sai lam do, bang each hue g dan HS tu nhan ra va sua chua, khac phuc sai lam" [105, tr. 70]. A.A Stoliar con nhan rnanh : "Khong duoc tiec thoi gian de phan tich tren gio hoc cac sai Cam cua HS" [106, tr. 91]. G. Polya da noi : "Con nguoi phai biet hoc o nhung sai lam' va nhiing thieu sot cua minh" [53, tr. 204]. B. V.Gonhedenco khi neu ra 5ph~m chatcua tu duy toan hoc thi da co toi 3 pharn chat lien quan toi viec tranh cac sai lam khi giai roan : . 1) Nang luc nhin thay duoc tinh khong ra rang cua suy luan; thay su thieu cac mat xich can thiet cua chung minh;·2) Co thoi quen 11 giai logic mot each day du: 3) SI:1 chinh xac cua li luan [98, tr. 31-34].

Chinh tu cac yeu cau cap bach va nhan thuc tren day; chung toi del chon dt! tai nghien cuu luan an la :

"Re n luye n nang hrc ghii toan cho HS PTTH thong qua viec phan . tich va su-a chira cac sai lam cu a HS khi gial toan",

'1 MVC otcn NGHIEN CUU.

Nghien cuu cac sai lam pho bien cua HS PTTH khi giai roan, dong thoi de xuat cac bien phap su pham de han che Val sua chua cac sai lam nay, chu yeu qua phan man Dai so - Giai tich nham ren luye n nang luc giai toan cho HS va gop phan nang cao chat luong d9-Y hoc man toan trong cac truong PTTH.

3. GIA THUYET KHOA HOC.

. - :

Neu cac GV toan 0 truong PITH nam bat dUQC cac sai l~m pho

- I

bien cua HS khi giai toano d6ng thoi biet each phan tich va su dung

cac bien phap day hoc thichhop de han che, sua chua cac sai Iftm nay thi nang luc giai toan cua HS se duoc nang cao hon, tu d6 chat IUQng giao due toan hoc se tot hone

-+. :"i~rrE:vl vu NGHIEN cCU.

Nhiern vu nghien cuu cu . luan an baa gam:

-+.1. Dieu tra cac sai lam pho bien cua HS PTTH khi giai toano

-+.2. Phan rich cac nguyen nhan sai larn cua HS khi giai toano

-+.3. D~ xuat cac bien phap su pharn voi cac tinh huang dien hinh

de han che, sua chua cac sai lam cua HS PITH khi giai toano

-+A. Thuc nghiern su pham de xern xet tinh kha thi va tinh hieu qua cua cac bien phap duoc d~ xuat.

). PHUONG PHAP NGHIEN ctru.

5.1. Nghie n CU'1l ly luan : Co so ly luan v~ tam ly hoc, giao due hoc, ly Iuan day hoc mon toan, dieu khien hoc, thong tin hoc de phan . tich cac nguyen nhan va xay dung cac bien phap day hoc nham han che, sua chua cac sai lam cua HSPTTH khi giai toano

5.2. D it~u tra tim hieu : Tien hanh tim hieu ve cac sai lam thong qua cac GV toan 0 nhieu vung trong ca nuoc, thong qua bai kiern tra true tiep HS 6 cac truong PTTH.

5.3. Thtrc nghie m suo pham : True tiep giang day thuc nghiern 6 khci to. 11. 12 truong PTTH dan lap Marie Curie (Ha N9i) trong cac narn hoc 1993-1994, 1994-1995, 1995-1996 va lop 1001 truong PTTH Chu Van An (Ha N9i) narn hoc 1995-1996. Chon lop 10 chuyen roan

. truong PTTH Le Hang Phong (Narn Ha) lam doi chung de xern xet tinh kha thi, tinh hieu qua cua cac bien phap da de, xuat.

6

6. NHUNG LUAN OrENI OVA RA BAO VE, NHUNG DrErvI iv[OI VA Y NGHIA THVC TIEN CVA LuAN /.\N.

6.1. Nhfrng lu~n diem dun ra bao v~ :

* Thuc trang dfmg 10 ngai ve cac sai llim cua HS khi gicii roan,

, .

doi hoi phai co cac bien phap cap bach, thich hop giup cac GV day

toan vuot qua.

z:

·'Cac dang sai lam pho bien cua HS khi gia: roan

: Cac nguyen nhan sinh ra cac sai lam cua HS khi giai toano IS the' • :~an che va sua chua duoc cac sai lam nay nho cac bien phap su pharn thich hQP·

6.2. Nhfmg diem moi va y nghia thtrc tien cua luan an :

- Luan an neu fa mot each co he thong cac sai Ifim pho bien cua HS PTTH khi giai roan thong qua 74 bai toan cua 12 loai roan trong phan men Dai so - Giai rich PTTH, cung voi viec phan tich cac nguyen nhan gay nen cac sai lam nay. Tac gia d~ xuat bon bi~.~_ phap s~_ph~rn_ voi ba phuong cham chi d90 sir dung trong cac tinh hu§ng s!ien_Q.lnQ_

..._ ---,·-·---... . 0_-.-

nharn han che va sua chua cac sai lam cho'HS PTTH khi giai toa!}.: __ I2~c_._

..... -~------.~.~--_-_..._-- --._------------_ ......•. -._---

~¢t lu~n an dua ra 8 dau hi~u d~. ren luyen cho HS t1;L nhan bjer loi-

giai co sai lam. Luan an g6p phan hoan thien them li luan day hqC:_E0:Y.Q

- ----' ._-_._--_ .. _----_._ .. -_- ' .. '-'---

toan 6 n LICC tao

-:_ Lu~n an cung cap mQt rai lieu tham khao bo i<;b __ de~QLduong.·,

___ 0 v toan, sinh vien 5U ph~m t()an g6£.J2h_an_. naQg_.cao hi~u -q~~-va--_--'hQc 6 cac truong PTTtL-

7 CAU TRU< LU~'-r .L\N.,

Ngoai phan 0116 dau, Kat luan va Tai lieu tham khao, Iuan an co 3 chudng :

'!' Chuorig I : Nghien cuu cac sai l'am pho bien cua HS PTTH khi giai toano

,~ Chuong II : Cac bien phap ren luyen nang 1I:1C giai roan cho HS PTTH [hong qua phan tich va sua chua sai lam.

--_

"~I CI1Lwng III : Thuc nghiern su pharn

Luan an co 10 so do. '10 bang va 1 phu luc,

7

Chu o ng 1.

NGHIEN CUD VE CAC SAl LAM PHO BIEN CUA HOC SINH PHO THONG

~

TRUNG HOC KHI OIAl TOAN

Theo Tl1 dien Tieng Viet thi sai ram la "trai voi yeu cau khach quan hoac voi le phai, d~n de'n hau qua khong hay." [95, tr. 830], phd bien la "co tinh chat chung, c6 the ap dung cho ca mot tap hop hien tuong, su vat" [95, tr. 775].

Chung toi hieu va su dung thuat ngil sai Iiim phd bien cua HS khi giai totin voi y nghia la : dieu trai voi yeu cfru khach quan (yeu cau bai toan ) hoac Ie phai (khai niem, dinh nghia, tien d~, dinh li, quy luat, quy rae, phuong phap suy luan ... ), dan tai khong dat duoc yeu cau cua viec gia{ COlan rna nhiing dieu nay xu at hien voi tflU sO' cao trong lei giai cua nhieu HS.

Can phan bier sai voi s6t. C6 khi s6t la sai : s6t dieu kien, sot rruong hop. Nhung c6 khi s6t chi Ifl thieu hoac chua d'ay du : s6t viec tong hop ket qua, rut gon ket qua.

V6i each hieu tren, chung toi da nghien cuu cac sai lam pho bien cua HS PTTH khi giai toano

1.1. TtNH Hi~JH ,tH1,fC TE QUA CAC DIEU TRA VA QUAN SAT.'

1.1.1. D ieu tra ttl' giao vle n : [66, tr. 23]

Chung toi gui phieu dieu tra ve cac truong PITH 6 nhieu tinh, thanh trong ca nuoc. Doi ttrang ghi phieu drill tra la GV dang giang day chuong trinh toan lop 10 6 cac truong PTTH Tien Du (Tien Son,

8

I .-

i :+. Khong logic trang suy dien

! 5. Hieu sai de toan

I
% v kiilz dong ~v
38,5
73,0 I
JtJ
77,0
I
I 61 < .. 1<:J
I .,-'
":+ ~ <, I
.J' D
,
69~2
50,0 . \
I
I 6- ., dt=1
I ),..)
·1
I·, I-in Sac), Bim Son (Ha trung, Thanh H6a), Le Quy Don (Ha Dong, Ha

Tay). Huynh Thuc Khang (Vinh, Nghe An), Luong Van TIJY (Ninh Binh), ~guyen Hue (Ha Dong, HaTay), Quang Trung (Dong Da, Ha NQi), Le Hong Phong(TP Ho Chi Minh), Nguyen Trai rvo rs«. Thai Blnh), t~e Quy Don (Long An), Phan Dang Luu (Yen Thanh, Nghe An).

I

Cac truong tren 0 nhieu vung dan cu khac nhau va dac diem khac

nhau : truong trung hoc, truong trung hoc chuyen ban, truong chuyen, Thai gian nhan phieu dieu tra ttl 30.3.1995.

':: Muc dich cua dieu tra : T1m hieu nang luc giai toan cua HS lop 10 PTTH. Tuy nhien, phieu dieu tra cling con quan tam toi nhiing muc dich phu nhu muc dQ chuong trinh toan hien nay 0 cac truong PTTH, nhung kho khan cua GV ve phuong phap va tai lieu tham khao, nhung bien phap rna GV thuong su dung de day giai bai tap toano Qua dieu tra nay, cluing toi nhan thay HS con pham nhteu sai tam khi giai Loan va moi doi tu ang HS dell co the mdc sai {am khi giiLi toano Cac GV''duoc hoi y kien ve nhung nguyen nhan dan toi sai 15m cua HS khi giai toan da eho biet (bang 1) :

Nguyen nhtin sai lam cua HS

: 1. Khong hieu khai niern, ki hieu

1 , .-

i 2. Tinh roan nham Ian

! 3. X(~t thieu truong hop

I

,

i 7. Nho sai cong thuc, tinh chat

I

! s. Di~n dat kern

9 '

'-

\ 'I

1.1.2. D ieu tra ttl' hoc sinh:

Ngoai cac dieu tra thuong xuyeno cac truong Marie Curie, Chu Van an] Ha NQi) rna chung toi co dieu kien true tiep giang day' trong cac narn hoc 1993-1994. 1994-1995, 1995-1996, chung toi con tien hanh dieu tra roan bo HS truong PTTH chuyen va chuyen ban Le Hong Phong (Nam Dinh, Nam Ha) vao cuoi nam hoc 1995-1996. Chung toi quan tam toi lop 10 gorn 13 lop, ,464 HS. Truong t6 chuc coi thi nghiern nic va dong vien HS lam het sue' minh.

D~ dieu tra thuc hien trong 90 phut :

Cau 1 : Giai phuong trinh

~ x2 - 1 - \j x + 1 = x + 1.

Cau 2 : Tim m sao cho phuong trinh

x - ~ 1 - x2 = m

co nghiern duy nhat.

Cau 3 Chung minh voi moi so a, b, c ta co :

(a- + b2)(bZ + CZ)(c2 + a2) ,?> 8a~2c: \:/

Can 4 : Tuy thea a, hay tim gia tri nho nhat cua bi~u thuc : / .I' F = (2x + y + 1)2 + (ax + y +3)2.

Luu y la HS .vua thi hoc ki xong nen roan bo kien thuc lop 10 da duoc on tap ki. De thi vua sue va rat chu y toi cac ?bay" sai lam. Cac sai lam rna dieu tra quan tam la :

* si : { AB 2: 0 va A 2: O} ~ { A 2: 0 va B 2: O} (diu 1)

'" S2 : Chuyen bai to an sang bai toan kh6ng tuong duong ( ~, ? ~ 1)

\ cau _, cau, .

",

- ,

. /" . . .., . . "'" '\

*S3 : Giai sal cau 4.

"- .... __ ... _- . __ ._ •..... ---_._-----_._----/

~,

\.

. A

*S4 :{A ~ B'va ;VI ~ N } ~ A.Nf 2: BN (cau 3).

*S5 : Tinh toan nharn lfin.

(

to

()

Duoi day la thong ke so HS mac.cac sai l'am tren (bang 2) :

-----_ Sai lam I Sl S2 S3
----- S4 S5
Lop --- -_______ I
-'
10 ch uyen Tin 2 14 12 19 31.
10 H6a Sinh .... - 37 41 34 42
.J)
: 10 chuyen Ly 10 ? .... 32 14 36 I
_.J
I
10 chuyen Phap 34 39 40 8 40 I
.~
10 Van Anh 30 46 54 1 ~ 54
10 Toan tin 18 29 26 13 40
lOch uyen Anh 9 30 ........ .., ....
.J.J .J')
10 Toan Ly 10 26 30 25 l-
~)
10 chuyen Nga O. ·4 16 4 16
10 chuven Tozm 6 14 / i~ \ 4 l6
( :- /
<::::::7
10 chuven Sinh 19 30 ·28 12 .., ....
.J.J
10 chuyen Van ..,,., ........ .. ...,..., 25 .., ...
.J.:.. .J.J ,.J') .J.)
10 chuyen H6a ) 19 22 6 31
! T6ng so HS I
210 344 379 195 455 I'
I
, 010 HS sai lam 46 82 1 42
75 I ' . .9.,9.
I . 1.1.3. Quan sat ket qua thi D al hQC :

Truong Dai hoc Sl1 pharn Vinh co nhiern V\1 tuyen sinh' ~h~{-teuto

cac tinh Nghe An, Quang Binh, Thanh H6a. De thi cua truong bam sat 1 chuang trinh pho thong. Ket qua thi mon roan cua cac HS thi VaG khoi A trong 5 nam lien tuc the hien 0 bang thong ke sau (nam 1990 khong I thong ke so bai duoi diem 5 - tai lieu do Phong dao tao cung Cap) (bang 3)- :

11

Nam

1989

1992

1993

1990

0.0

.,

.)

i .

8

' .

i

13

, I ( i - I
i
1,0 I 57 I 116 I 31 I 16
I
: i
I I !
1,5 i 67 70 36 13
I
I
I
, i I
I 70 I 60 98 55 13
: -, I
- ! ~ I 05

91

86

15

2,)

63

34

18

3,0 74 58 32 25
!, 3,5 73 54 41 27
+.0 54 )) 44 21
.:I. ~ 74 .:1.3 41 28
5.0 65 37 26 21 43
5,5 42 38 20 22 31
6,0 -1-5 26 21 12 34 I
6,5 I 24 28 16 16 20 i
i \.
7,0 24 18 12 11 14 I
I
7.5 18 19 07 07 14
8,0 11 14 06 06 02
8.5 07 04 04 02 02
9,0 02 03 04 02 02 - ..
l) - 00 01 I 02 i 01 I 01
I i i I
i ~ ,) i I I
I I
i 10,0 I 00 I 00 I 00 I 00 I 00
I ' ,
I T6na , 854 I 365 I 442 I 289 I
I
!:I I I 1
I i I !
.. , '1 , '" So %HS ttl 5 tro len

27,9

.:..1,7

(188)

26,7 (118)

.)4,6 (l00)

(163)

(238)

12

l.lA. Theo d6i cac bao cao khoa ryQc :

Trang HQi nghi "06i rnoi phuong phap day he Toan 6 truong pho thong trong giai dean hien nay" tai Vinh (20 - 21/12/1995), chung toi quan tam tai 2 bang thong ke sau day;

a. K~t qua thi thu tot nghiep rnon Toan 1995 (de cua BQ GD DT) cua HS truong PTTH Huong son, Ha tiuh [58, tr. 124] (bang 4) :

Diem

o den 2

5 den 7

8 den 10

So HS

80

120

40

10

Nhu v~y so HS trung binh tro lenla 20%.

b. Ket qua kiem tra chuong II, DS 10 cua truong PITH chuyen ban

._. "-"".-"-~'-------,-.;..--.--

Thanh Chuong I, Nghe An : [58, tr. 54-56] (bang 5) :

Doi tuong HS

Tv 5 tro len

Chu v them

Leo chon ban A

L '

65%

Khong c6 diem duoiJ

,--

I

Lop thuang ban A 30%

i Ban C 13,3c.;'O 13,3% diem 0 va 1

L_ ~~ ~ ~

1.1.5. Nhfrng ket lu~n can thiet

" HS con mac nhieu sai Cam khi gia: toim,_~~,_~_ae ~op'~_~uye~_~_ nhiing trucng eo truyen thon~ hoc tot, day tot.

---.--------.-_----------:r---:------ -------------- .... ,_, ... -'-

';' Viec Iinh hoi cac khai niern toan hoc ella HS, dac bier la cac

khai niern rnoi duoc dua vao chuang trinh pho thong con gap nhieukho khan, rna doi khi lai xuat phat tu .su lung tung v'e phuong phap day hoc ella GY.

, ,

,* Nhieu GY chua luu y cho HS nhiing sai lfim c6 the m~c plrai

khi giai roan.

13

1..+

:., S u can thiet phai co mot nghien cuu nghiern rue ve cac sai Cam cua HS khi giai wan tren cac phuong dien : the hien, nguyen nhan, ngan ngua, :,;,hac phuc de bo sung va hoan thien vao phuorig phap giang day man Toan. nang cao hieu qua cho viec day hoc Toan.

\ I

'2. ~H)T s6 SAl LAN[ PHO BIEN eVA HOC SINH PHO THONG TRUNG HOC KHI GIAI TOAN DAI so - GIAI TICH

Trong luan an nay, chung to: khong d~t nhiem vu thong ke het moi sai fam cua HS PTIH' khi giai toan, rna chi nell len nhung !ai [am phd bien ella HS klu; gioi, day la nhting sai Iflrri rna qua nghien cuu, chung toi nhan thay co tan so cao trong cac loi giai toan cua HS. Nhung sai lam nay co khi kha tinh vi, rna nhieu khi GV kho phat hien kip thoi, Viec dua ra cac sai Cam pho bien nay nharn rae dung ngan ngua cho HS va GV khi giai toano Kern thee lei giai sai (ki hieu ':) chung roi con phan rich sai Cam cua lei giai (ki hieu l) giup cac d6ng nghiep tharn khao de xu li cac tinh huang khi HS giai sai. Viec chi ra mot loi giai sai khong phai hie nao cling de dang. Day cung la nhung co gc'ing ella chung toi gop phan nang' cao hieu qua day hoc man toano De thuan 10i cho viec thea d6i. chung toi xin trinh bay 7-+ thi du thea l2 loai roan

,

thuong gap trong chuong trinh Dai sc - Giai rich PTTH. Cac loi gia: dung cho cac thi du chung toi da trinh bay trong [6]. Tuy nhien voi rnot vai thi du can nhan manh, loi giai dung van dUQC dan ra.

1.2.1. Sai Him khi bien d5i bieu thirc :

Nhilng sai lam khi bien d6i bieu thuc thuong mac khi su dung d.: dang thuc khong phai la hang ruing thuc, do la cac "a hang dang" - ch dung voi nhung dieu kien nao d6. D6i khi sai lam xuat hien do hiet nharn cong thuc.

Sai

': N goai 3 thi du tren, HS con hay mac sai Ia.m khi su dung cac cong .huc bien d6i sau day (bang 6

Dunz

e

. .

23" _ 2(3) _ 29

,

1,3- = 23. Z = 26

1 1

10,& 3 = - -----

x loa 4x 2100' 2 + 100'- X

~ !:13.:::0:3

Ie

1

= loa. 3 = -loa 3

°z-. x 2 :;:'2:<

" ;: - ?l . 2

., X - _ Og, X

-~ -~

0;

~---

I

I 2.2·~=2 1-:(

') ').'( = ,·f

i (fa + ~ b)l = a + b + 2 ~ ab

! voi a > 0 va b ~ 0

i

1.2.2. Sai lam khi giai phuong trinh, bat phuong trinh

Nhung sai f~,al khi giai phuong trinh thuong mac khi HS vi pharn

quy rae bien (f .'. h3Y rhieu c5.L\.dit· mL1C ... kh6ng gil viec bien ct6i

chuang nay.

Thi du 1. i 3~ ') C)¢,

"\s trinh, bat phuong trinh tuong duong. Dar thua d~u d~n den rihiing sai lam, tham chi sai den ·.luQC rnla ! MQt s6 sai lam con do hau qua ella .. tic khong dung, nhu da ehi ra 6 muc 1.2.1 ella

iuong trinh ,

- 9x = 9 (X2 - 2x - 3)

.J. - 2x - 3) = 9 (X2 - 2x - 3)

(*)

¢> 3x = 9 ¢> x = 3.

16'

! Co the thay ngay x = -1 cung 1ft nghiern. Sai Iftrn la HS da chia hai ve ella phuong trinh eho XZ - 1x - 3 rna quen ding:

ab ;:= cb ~ b(a - c) = 0

Thi du 2. Giai phuong trinh

~ -xj + 3x - 2 + "JX + 1 - ~2 ? Dieu kien can thuc co nghia :

{-x3 + 3x- 1 ~ 0 .:::..{X3 3x + 2 :5 0

x + 1 ~ 0 x ~ -1

Q {(.X - 1)2 (x + 2) :5 OQ {xx + 2 :5 0 Q {x :5 -2.

x ~ -1· ~ -1 x ~ -1

Vay khong ton tai gia tri ella x de hai can thuc d6ng thoi co nghia nen phuong trinh vo nghiern.

! Co the chi ra voi x = 1 thi ca hai can thuc deu co nghia va x = 1

• j

chinh la nghiern ella phuong trinh. HS del sai khi giai bat phuong trinh

(x - If (x + 2) :5 0 Q X + 2 :5 O.

Thi du 3. Giai phuong trinh

~X2 - 1 - ~x + 1 - x + 1

,) Dieu kien can thuc co nghia :

Jx:: -1 ~ 0QJ(x - 1)(x + 1) > 0

lx+.1~O lx+l~O

{X - 1 ~ 0 {x ~ 1

.:::. Q =- x zi !

x + 1 ~ 0 x ~ -1· -.

Khi do phuong trinh co d~ng

~ (x - i) (x + 1) - ~x + 1 = x + 1

Vi x ~ 1 nen ~ x + 1 > 0, chia hai ve cho ~ x + 1 ta co : ~ x-I - 1 = ~ x + 1.

Vi voi x ~ 1 thi ~x - 1 < ~x + 1, nen ~x - 1 - 1 < {X +-1. 1

V~y phuong trinh vo nghiern.

17

Sai l'am khi giai he {:' +- [I : eO' nhieu HS tuong rang i A ,B 2= 0 ¢:> {A 2= 0,

LA 2= 0 B 2= 0

6 loi giai tren thieu x = -1 va d6 chinh la nghiern duy nhat cua A = 0

B eo nghia A > 0

B ~ 0

I

f .. I·/(/le

/

, h HS d~ " ~ {A. B ~ 0

phuong trin . a quen rang A 2= 0 ¢:>

Loi giili dung to. : Dieu kien din thuc eo nghia

{x':: - 1 ~ 0 f [x ~ 1 1 [x-

1 O¢:>~ X$;- ¢:>

x + > l 1 x 2= 1

x 2= -

-1

Thay x = -1 tho a man phuong trinh Voi x ?: 1 lam nhu loi giai tren,

T6m lai : phuong trinh eo nghiern x = -1. Thi du 4. Giai va bien luan phuong trinh

. ? -

~ ... a +,) 0 ( . h h ... '

a - ,). + ,=. "') t eo t am so a.

x - .:.

') Dieu kien : x ;t; 2. Kill d6 (*) ¢:> (a - 5) (x - 2) + 2a + 5 -:- 0

¢:> (5 - a) (x - 2) = 2a + 5 I

¢:> x (5 - a) = 15

15 N e u a ;t; 5 thi x = 5

a

:, .... ~{ ... ,i If

at" fl.{, I.! ";', v -'- ~\.,~ . I ,

i

,

" /

Neu a = 5 (hi phuong trlnh va nghi~m. \.'1'1;

»: 15 khi , khc l' ~h' " '~ ~ (

! ~ai lam la HS khong d~ y x = _ 1 nao ong a ng tern YI(l .

-- __ ,) __ . __ .a_--.- __ ...-. _ _ ..

cua phuong trinh. Vi nghiem phai thea man, x ;c 2 nen khi

15 5

= 2 ¢:> a = ? thi phuong trinh vo nghiem. Loi giai phai b6

5 a _. . ,-

sung dieu nay, va ket luan dung fa :

18

.' ..

",

t':: ••

{a ;c 5

Neu a;c 5

2

Neu [: = 5 5

2

15

thi x -

5

a

thi phuong trinh vo nghiern

Thi du 5. Giai phuong trinh

2x + ~x - 3 = 16

(*)

? Di~u ki~n : x 2: 3. Ta co

(*) <::> ~x - 3 = 16-2x ~ x - 3 = 256 - 64x +4X2

l-x = 7

<::> 4x2 - 65x + 259 = 0 ¢> x = 3;.

~'7

_, ,

thea man x 2: 3. Vay phuong trinh co 2 nghiern x = 7 hoac x _

-+ .

( ! Sai lam khi viet ~x - ~ = I? - (2x ,¢>; x_- ~ _,.256 - 64x + 4xz,

Can luu y HS ding:

{b 2: 0

fa = b¢>· a = b-

"'7

(khong can d~t dieu kien a 2: 0). Ta co x = .)4" khong la nghiern

Thi du 6. Giai phuong trinh

log, x2 = 2logz (3x + 4)

....... {x ;c 0

-- 4

Ox> - .., .

.)

(*)

') D'" k" {XZ > 0

leu ien : 3x + 4 >

Khi do C') ¢> 210gz x = 21og2 (3x + 4) ¢> log, x= log, (3x + 4)

¢> x = 3x + 4 ¢> x = -2. '

19

Gia tri nay khong thoa man di~u kien del dat nen phuong trinh vo

! Sai lam k h i bien d?i log, x2 == 210gz x, do do mat nghiern x - - 1.

Thi du 7. Gia! phuong trinh

·'IOg,.:'( + 6 0

'+ X - ==

') Ta co 41o~:'( - (210~)2 - x2 nen phuong trinh tuong duong voi x2 + x - 6 == 0 ¢:> [x _ ~ 3

x - _.

! C:3n luu y HS : 2Iog~:'( - x chi khi x > O. Do do chi lay duoc x == 2 la nghiern.

Thi du 8. Tim m sao cho phuong trinh

19 (X2 + 2mx) - 19 (x - 1) == 0 (*)

co nghiern duy nhat.

? (*) ¢:> 19 (X2 + 2mx) == 19 (x - 1) <=> x- + 2mx == x-I

¢:>x'-:+(2m-l)x+l == 0

(* *)

,-

Phuong trinh c6 nghiern duy nhat khi ~ == °

¢:> (2m - 1)'-: - 1 == 0 ¢:> 4m2 - -lm == O· <::> [m == --0-m=l. ~

•• ,1IiI..

- ! Trong nhieu ki thi dai hoc, loai phuong trinh nay xuat hi~n lam kha nhieu HS giai sai. Tren day chi Ia mot ki€u sai. Sai l~rn la khi nul hoa kbu logarit. HS da khong dat dieu kien nho duoc log, f(x) == log, g(x) fO<a;r:l

¢:> 1 f(x) = g(x) (chu Y f(x) == g(x) nen chi d~t dieu kien f(x) > 0,

f(x) > 0 khi f(x) don gian hon g(x)).

20

/

[

M9t so c6 luu Y toi dieu kien x > 1 nhung lai li luan : (*) co nghiern

. (il = 0

duy nhat "" ( .. ) co nghiern duy nhat x > 1"" t; > 1 ( 0) .

A ..- • ~ ~ .'~ • ~ {xz + 2mx > 0

M9t SO lai dC:lt he dieu kien X .: 1 > 0

va I vap phai be' tac kh6ng giai tiep duoc mia !

Thi du 9. Giai bat phuong trinh

1 1
~X2 2x ., <
- - :; x + 5
¢> x + 5 < ~X2 - 1 - 3
.;.x (*)

? (*)

¢> (x + ;))Z < .'\2 - 2x - 3 7

¢> 12x + 28 < 0 ¢> x < - 3

, HS . I'A khi hi r] ;' 1 . 1 b

. Sal am 1 ng 1 ranW a <. b ¢> < a,

~---- ;;-:-~---

, dii 1

rna uris ra - < :::> a

1 a - b

-¢> > 0 ¢>

b ab

a >b ab < 0 a < b

L

Thi du 10. Giai bat phuong trinh -1

e

(*)

? Ta co flex) = x va f~(x) = eX lit cac ham so dong bien tren R =:> f(x) - x. eX lit tich cua hai ham dong bien nen ciing d6ng bien tren R.

Ta c6

. -1 t(-I) = (-1). e-1 = -

- e

Do do (*) <:>

f(x) > f( -1) ¢> x > -1.

21

x 1- 00 - 1
.-
f'(x) I 0
f(x) +OO~
1
e +00

L/

7 /

/// ~

I l

'[

Sai l'am khi nghi ding rich cua hai ham dong bien la ham dong bie'n Neu cac ham dong bien chi nhan cac gia tri duong thi m6i duoc ket luan nhu v~y.'

Loi giai d u ng ia :

Xet f(x) = x. eX v6i X E R ta c6 f'(x) - e-(x + 1) nen bang bien thien cua f(x) la :

+

1

Ttl d6 ta c6 f(x) > - - <:> X :;z: -l. e

1.2.3. Sai lam khlchung minh bat dang thirc

Cac sai lam thuong bat nguon tu viec van dung cac bat dang thuc c6 dien rna kh6ng de y t6i dieu kien de bat dang thuc dung,_~~~c su dung sai sot cac quy rac suy luan khi ttl bcit dang thuc nay suy ra bat

.... _ .. _- - .-_" ..

dang thuc kia.

Thi du 1. So sanh

1

x + va 2

x

.) Ap dung bat clling thuc Cauchy cho hai so x va ~ ta co

1 ( x ~) 0 1
+ > -
2 x
1 2
=>x + ~
x I')

-_

Dang th uc xay ra <:> X

1 .~ 1 1

- - <:> x- = <:> X = ::!: •

x

Sai Iflm VI khong de y dieu kien cua cac 50 a. b trong bat dAng thUC Cauchy:

a + b

2

voi a ~ 0 va b ~ 0

Thi d1:L 2. Chung minh ding voi moi a ta co :

a(l - a) s ~

(*)

? Ap dung bat dAng thuc Cauchy cho hai so a va 1 - a ta co : a+1-a 1

~ a(1 - a) ::;

2

1 a(l - a) s 4

! Lai v<1n sai nhu da phan tich 6 thi du 1, vi a va 1 - a chi khong am khi a E [0; 1]

Lui gilli dung La :

c:)

, 1, 1 0

~ a - a~ ::; - ~ a- - a + 2:

4 4

( 1 \ ::

~ a - -) >

\ 2

Ghien nhien dung voi moi a.

Thi du 3. Chung minh rang neu x > -1 thi x . e''( >

-,

~

e

.) Ta co x > -1 (1)
e-1 1 (2)
ex > _ ..
e Nhan thea tung ve ella (1) va (2) ta co : 1

x. e''( > - - (dpem). e

Can luuy IjS ding :

{~ : ~ :

o

o ~ ac > bd.

Khong the nhan tung ve cua (1) va (2) d,e suy ra dieu phai chung minh.

Thi du 4. Chung minh ding: Neu x 2: y > 1 thi

> v + E.

J I J

/

') V oi x > Y > 1 ta c6 :

x 2: y va fX 2:' {y-

'!

'no thea tung ve ta c6 x - fX 2:

! Sai lam khi tru tung ve cua hai bat dAng thuc cung chieu. Cftn

I _

luu v HS :

"

t

Lui giai dung La : Xet f( t) = t - {f voi t > 1 ta co

{a > b

c ;: d f a - c 2: b - d.

f'(t) = 1 - 2~ - Do d6 f(t) dong bien voi t > l. Ma x 2: y > 1 nen f(x) 2: fey) => x - ~x > y

2ft - 1

,).r; > 0

.:...~t

~y =>

x + ~y 2: Y + -{X (dpcm).

Thi du 5. Chung minh voi rnoi x > 0 thi sinx < x. ? Xet f(x) =·x - sinx voi x > 0

Ta co rex) = 1 - COSX,2: 0 => f(x) dong bien voi x > O.

_" ---------_' ----, .......•. _._ .. " .. _ ... ,_'_.-._. - '/\,-... - .

t_Tu x > ( u ra f(x) >. f(O

"------

:;> x - sinx > 0 - sinO = 0

! Lei giai c6 ve dung, nhung sai lam kha tinh vi ~ Sau khi ket luan f( x ) dong bien tren (0; +:0) thi vi sao ttl x > 0 suy ra ducc '!1~}__-? f(O). ,Cfm hru y HS ding: ?'feu f(x) dOng bien v6i x E [a; b] va

.r:

'a < Xl < x2 s b thi f(xJ > I(x.). 0 ~di giai xet f(x) dong bien tren

. .

mien (0; -too) khong chua so 0 nen khong th~ so sanh .f(x~ v~ f(O), khi x > O.

24 -

a= + b96 + e9l> 2: 3 ~(abeto a95 + b95 + e'J5 2: 3 ~ (abc)"

(1) (2)

Thi du 6. Chung minh ding: Neu.

{a + b + e > 0 (1) ab + be + ea > 0 (2) abc > 0 (3)

thi a > 0; b > 0; e > 0

.) Do vai tro binh dAng ella a, b, e nen ta chi can chung minh a > O.

Gia su a < 0 thi ttl (3) => be < O.

T u (2) => a( b + c) > - be > 0 => b + e < 0

Tu a < 0, b + e < 0 => a + b + e < 0 rnau thuan voi (1). Do do a > O.

! ~?i_phll_ dinh a > 0 de thuc hien phep chung minh ph~~ efiup~ thi phai xet a ::; O. Lei giai tren thieu truong hop a = O.

Thi du 7. Chung minh ding neu a, b, e > 0 thi : a96 + b96 + e%

'>5 b~5 95 2: 3.,J abc.

a + + e

., . Ap dung bat clling thuc Cauchy cho 3 so duong ta co :

Cac ve ella (I) va (2) deu duong nen chia theo tung \'C ta duoc

(abc)" ~ abc.

(abc)~~ -

! Suy luan tu A 2: B >0 va C > 0 > 0 de e6 ~ 2:1 ~I 1a sai, Chang han 3 >'1 va 9 > 2, nhung kh6ng suy ra duoc ~ > ; ("),

25

a+b+c

(1)

Lui gitii dung La :

Do vai tro binh dang cua cac so a, b, c nen co the gia su

n < a < b S e =:> a95 < b~5 < C95• Ap dung belt dang Tehebyeheff ta co : 3(a.a95 + b.b" + c.c") 2: (a + b + e)(a95 + b95 + e95)

Ap dung bat dang thuc Cauchy ta co

a + ~ + e > l.J abc (2). Tv (1), (2) suy ra dieu phai chung rninh.

~ .'

26

1.2.4. Sai lam khi tim gia tr] 16n nhfit, gia trj nho nhat

Nhiing sai lfim khi tim gia tri Ion nhat va gia tri nho nhat ella ham so hay cua bieu thuc nhieu an thuong do vi pham quy t~e suy luan

logic:

" N e u f ( x) ~ m (m 1 a han g so), v 0 i m Q i x E A vat 0 n t a i x') c: A sao eho f(xo) = m thi gia tri nho nhat cua f(x) tren mien A la mil (eo quy [aC tuong tu eho gia tri Ion nhat cua f(x».

Doi voi bieu thuc nhieu an, cung e6 quy uic tuong tu, Thi du 1. Tim gia tri nho nhat ella :

Frx, y) = (x + yf + (x + If + (y - 2)2

'") V oi moi x, y E R thi : (x +y)2 ~ 0 (x+l)2 > 0

(y - 2)2 > 0

Vay F(x, y) > 0 'r/x, i E R. Tv do suy ra minF(x, y) = 0

! Sai Cam ella loi gia] la khong chi ra cac gia tri cua x va y d~ F(x;y) = O. Nho ding: F(x;y) ~ 0 'r/x, Y E R va neu ton tai XI); YCl sao

x =

.,

.J

..:hn F(x,,:Y,,) = 0 thi rnoi ket luan duoc min F(x:y) = O. Dei voi bai

.

roan nay. khorig ton tai xu; Yu d~ F(xo;Y 0) = O.

Lui giai dung La :

Ap dung bat cliing thuc Bunhiaeopski voi

a1 = -1; ~ = 1:

a~ = 1

.)

b, = (x + y);

b2 = X + 1;

ta ~6 :

1 = I(-l).(x+y) +1.(x+1) +1.(y- 2)1

s {j. ~ F(x, y) =:> 1 s 3F(x; yJ =:> F(x; y). ~ ~

{'"'X + v = -1

<=> : - ; = -3 <=>

y - "'

.J

,5

1

Vay : minF(x:y) = - <=> X = 3 .

+ 5

""y-.,.

.J .J

Fill d~t"':. Tim gia tri nho nhar ella

.(x) , 1 ,.., I 1)

t x . = X" + -, - ~ \ x + -

. X" \ X

+.5

,) D '. 1 hi 1 , ')'

. at [ = X + - t 1 X" + --:; = tN' - _ nen

. X X"

f(x) = g(t) = t'! - 2t + 3 = (t - 1)2 + 2 ~ 2 V''t E R. Dang thuc xity ra <=> t = 1.

Do do : min t(x) = 2 <::> t = 1.

~ .~~~lam_,~a ~.!!~~~_n __ ~~.L~2~!1,._k!?-6n.g_~':l_()I_1g("!uong. 9ia trj nho 'nhat ella f(x) khcng trung voi gia tri nho ,nhat ella g(t) voi t E R. C6 the

thay ngay voi t = 1 thi khong t6n tai x va thuc fa sai lam 6 lei giai nay lai tro v~ sai lrun 6 thi du 1 vi khong eo gia tri cua x d~ rex) = 2.

27

28

Thi du 3. Biet rang x'; + y'; - x + y. Tim gia trj Ion nhat cua

F = xY·

.) Ta co (x - y)2 ?! 0

=:> X Z + i ?! 2xy

\fx,y E R \fx,y E R

\fx,y :: R.

D~ng thuc xay ra ¢> x = y.

Thay x = y vao he thuc da eho ta co 2xz = 2x.

¢> [x = 0

x = 1

l)-Ieu x = 0 thi y = 0 nen F = O. Neu x = 1 thi Y = 1 nen F = 1.

Tu d6 suy ra maxF = 1 ¢> x = Y = l.

! Can luu y HS ding: F S M voi M la hang so thi khi ton tai x. y de F = NI, moi ket luan maxF = ·M. Lei giai tren sau khi chung

minh F s x.;; {- da eoi x'; ; i nhu la hang s6 (!) va mac sai lam !

Thi du 4. Tim gia tri nho nhat cua

1

f(x) = (X + ~ ,.,

x + .J

.r::- '"' 1 '"'

'l '" + j +. ., - .J.

\jx + .)

Ap dung bat dang thuc Cauchy cho 2 so duong

-

'? Ta c6 f(x)

·1, {X + 3 va {X ." ta co +.J

{X+

voi rnoi x ?! O.

Dang thuc xay ra khi {X + 3 :::; {Xl.., ~ (~x + 3)2 = 1 +..)

Thay ngay khong co gia tri cua x thea man VI {X + 3 === 3 ~ (~x + 3)Z === 9 > l.

Vay f(x) khong co gia tri nho nhat.

! Kh6ng co gia tri cua x de f(x) = -1 thi chi suy ra dUQC gia tri min f(x) > -1 va loi giai tren kh6ng di de'n duoc minf(x).

Thi du 5. TIm gia tri nha nhat

fOe) , I _' + "+') +'1

x = ~ x- - ..:.x 'J x- _x .:...

?Ta co f(x) = ~(x - 1)2 + 1 + ~(x + 1)2 + 1 Chon tren mat phang toa d¢ cac diem

ivl(x;O) ; A(l;l) ; B(-I;I)

thi f(x) = MA + MB.

Theo bat dang thuc tam giac thi :

f( x) = Y[A + iviB === AB = ~ (1 + 1 r + (1 - 1 r = 2.

Tu do min f(x) = 2.

! L:1i sai lfim vi khong de y co ton tai vi tri cua M de MA + MB = AB

hay khong, nen kh6ng ket luan duoc gi. 0 day: ?vIA + NIB = AB ~ M thuoc dean thang AB, nhung NI thi thuoc Ox, AB II Ox nen khorig ton tai ~L

Thi du 6. Tim gia tri 16n nhat va nho nhat cua cosx + 2sinx + 3

v =

2cosx - sinx + 4.

voi x E (-;r : n).

? Ta tim tap gia tri cua y, tuc la tim y de phuong trinh y = f(x) co nghiem.

Vi 2cosx - sinx + 4. > 0 Ttx nen y = f(x)

29

Vx

¢:> Zycosx - ysinx + 4y = cosx + fsinx + 3

¢:> (2 + y)sinx + (1 - 2y)cosx = -1-y - 3 Phuong trinh c6 nghiern khi a~ +b- 2: c~ ¢:>

(2 + y)2 + (.1 - 2y)~ > (4y - 3)2 ¢> 11 y2 - 24y + 4 s 0

? ?

¢:> 1-1 < Y < 2. Do d6 max y = 2 ; min y = 1-1 .

Loi giai quen han yeu cau x E (-n ; n). Le ra thea duong loi tim tap gia tri thi phai tim y de phuong trinh y = f(x) c6 nghiern x ~ (-.1: ; :r). '

Lui eiai dung la : Dat t = ta-! thi x E (-n . n) ¢>

.... . ~2 '

~ E ( -; ; ;). Khi d6 phuong trinh y = f(x) c6 nghiem

x E (-."1' ; :r) khi phuong trinh y

,

., ? ?

t .. + _t + - , hi A R

, .., co ng tern t E .

t .. - t +..) .

Til do co Y EO L21 ; 2] nen max y = 2 va min y = 121"

1.2.5.~~~_~~_~ ~~i J~.~~i. ~_~~.~a! to~~_.~~!!!.~_hti'C bac hai .

Khi giai toan tam thuc bac hai, cac sai lftm xuat hien do khong

. - -_._-- ---~ --- ---_ .. _. " . ---------_._-_ .. _ ... _ .. __ .

chu y den gia thiet cua cac dinh li rna dcl VQi vang ap dung hoac la lam

,_ _ _.- .. __ . __ .. __ ...• _--_ _ .. _--_._------- ---------_.- _-_ _ --, __ ....• -.- .. _.

dung suy dien nhung menh d~ kh6ng dung hoac xet thieu cac truong

...... -_ .•.....•.. , ~ .. ---- .. ,--- .. ,_._-------_,_ --_-------. -- -. -. -- - - ~ - --

/ . u ;.~ _I:; i ,<~ .. ; '"

- l~.: \.i~ .~ 1 t L'u. ~!_:---.- . ,,-"

..• _·····'r ,.1'---'

hop can ~i~~_~.~.Il ..

Thi du 1. TIm m de bieu thuc

~ (m + l)x" - 2(m - l)x + 3m - 3 co nghia voi moi x.

? Bieu thuc c6 nghia voi rnoi x

¢:> f(x) = (m + l)xZ - 2(m - l)x + 3m - 3 ~ 0

30.

Thi du 3. TIm m sao cho :

2 I +., +1

X - ..;.mx -'m - , 'W

s 1 voi vX E R. ? 2 + ')

_x - mx _

; (*)

fa > 0 {m + 1 > 0

¢:> l~':< sO¢:> (m - 1)2 -.3(m - l)(m + 1) s 0

{m > -1 . . r m[ > -1

~ 2( m - 1)( m + 2) > 0" 1 : ~ ~ 2 .. m 2: I

Ta c6 ket qua m 2: 1.

! Can luu y HS ding f(x) a - b - 0

c 2: 0

¢:>

a > 0

a: s 0

== ax- + bx + c 2: 0 Vx

Thi du 2. Tim m d~ phuong trinh

(rn - l)x~ + (2m - l)x + m + 5 = 0

. .

c6 hai nghiern phan bier.

? Phuong trinh c6 hai nghiern ph an biet khi ~ > 0 ¢:> (2m - 1)2 - -+( m - 1) (m + 5) > 0 21

¢:> -20m +21 > 0 ¢:> m < 20'

! C6 th~ chi ra voi m == 1 < ~~ rna phuong trinh chi c6 1 nghiern

. t

x = -6. Nho ding: f(x) = ax- + bx + G c6 dung hat nghiern phan bier fa ~ 0

¢:> i ~~ > o·

c

'? (:1<) ¢:> x2 - 2mx + 3m + 2 5 2x: - rnx + 2 'tix:: R

¢:> x2 + mx - 3 m 2: 0 VxE R

¢:> ~:< < 0 ¢:> m:! + 12m < 0

~ -12 S m S O.

31

{(2m + 1)2

¢> 2m + 1 '"

2 >,j

- 0 {4~1 . ~ _ 1 - 0

¢> )

m > - ")

D

! Sai Cam khi nhan hai ve voi 2X':! - mx + 2 khi chua biet dau cua

bieu thuc nay.

Tlzi du 4. Biet ding (x:y) la nghiern cua h~

r . _

lX + y - m .

. x:! + v:! = -m2 + 6

J

Tim gia tri Ion nhat va nho nhatcua F = xy - 6(x + y) .

. ? Ta co x2 + y2 = -m':: + 6 ¢> (x + y)2 - 2xy

= - m Z + 6 ~ m 2 - 2xy = - ml + 6 ~ xy = m ' - 3.

,

Do do: F = rn" - 3 - 6m

= m - 6m - 3 = (m - 3)2 - 12 Vav min F = -12 ¢> m = 3 .

• J

F khong co gia tri Ion nhat vi F la ham bac hai voi he so m- la

a = 1 > O.

! Lei giai khong dat dieu kien de ton tai x va y. Do do da xet F voi 'tm E R.

Thi du 5. TIm m sao cho phuong trinh : x2- (2m + l jx+ m' = 0 chi co mot nghiern thea man x > 3.

'- '? each 1 : Phuong trinh co nghiern duy .. n~at 'khi Cl~ = O. ih!l~(&t~~":,,, .. t.~,.~

phuong trinh co nghiem Xl = x~ = ; . Do do phuong trinh chi co Jt~

- -

. nghiern thoa man x >

32

r I n e-

To' lai . (5.., .-,:;- ]

10m ai : mE,:2 ; .J + ~.J

0.:. ,I

s I ( .". ,1' . ~ . \,:1A, I , ..... yt,l .. _A..

,.'

<>{:>5~

l :2

V~y khong co m thea man bai toan.·

I' .... , i,\. "

1\ 'I Y.

\, I. ,

i I

L I, \ 1.../

- .... ,

,

It I'

1""1

1\ I. 'I I -. I l

? each 2 : Xet hai truong hop

'. ....

\, .. ;~J!

',d \ .. \

5

m >

2

Kh6ng co m thoa man truong hop nay.

{a f(3) s 0

* Truonz hop 2 : Xl !S 3 < x, ¢> S ..,

'-' ~ ->.J

2

r m- 6m + 6 < 0 {: "" s :s .,
- \j.J m .J
¢> 12m -t-: I ¢> 5
> 3 > 2

) .., fl. /
¢> - < m < .J +
'1 ! Cach 1

~ . ~, !

I ,\ II : ' l" \;

I \ \A.~ I" I. ".

\\f-" .: '") ,):.~ ( ....... ~..J .: {

// i',' 'I ~, l I. ,\ l: ('/-.-I

\ L t j, v, _)

to ra HS chua hieu cum tu "chi co mot nzhiem" nen da

.• ~ ,~ "-! - •

"phien dich" tung doan yeu cau, thanh ra khac voi nghia cua bai toano Can hru y cho HS la phuong trinh chi co mot nghiem thoa man X > 3

I •

khong co nghia la phuong trinh khong duoc co hai nghiem ! Cach 2 la lei giai co gang lam gon hai truong hQP Xl < 3 < :s va 3 = Xl <l x, thanh mot truong hop ~ s 3 < :s. Tiee rang khi viet dieu kien "tuong duong'' voi yeu clu

, I i khc d' Nh ~ -b" xis h S.., Chi h '

nay ~I ' ong ung.t L1 v~y se a sot truong op Xl < 2 s .J < x;: . In VI

..,,..,

.J.J

vay rna voi m == 2, phuong trin~ tro thanh x:! - 5x + 4- == 0 ¢;> l: ! thea man bai toano nhung m = 2 khong co trong ket luan cua each giai th u hai.

Ket qua dung la : rn E (3 - {3; 3 + v] . ThE du 6. Chung minh ding phuong trinh

(x - 95)(x - 96) + (x - 96)(x - 97) + (x - 97)(x - 95) = 0 co hai nghiern phan bier Ion hon 95.

? GQi ve trai la f(x) thi

f(x) == 3x2 - 2(95 + 96 + 97)x + 95.96 + 96.97 + 97.95.

Do do : af(95) == 3(95 - 96)(95 - 97) > 0

va

S 9- - - .)

I

_ 95 + 96 + 97 _ 95 == 1 > 0

.,

.)

'Tu af(95) > 0 va; > 95 chua ket luan duoc f(x) co hai nghiern . , (I. -v -r l _j

. l' ';', " i, \

, ,1-".1.'"

phan bier.

Lei giai thieu cong viec chung minh quan trong nay.

1.2.6. Sai lam khi xet cac loai h~ phU'011g trinh, bat phuong trlnh, Sai lam khi xet cac loai h~ phuong trinh. bat phuong trinh thuong xuat phat tu nguyen nhan kh6ng narn viing cac phep bien ct6i tuong

_, - •. ,- -'---.- .. -.-.- __ . ._ .. ._._,_."._. __ , ,_ ,._, ,.,_ '_., N_. _ .. _

{' "'1 0

x- - .)x" + X + ==

, xJ - 4x2 +' 2x + 3 == 0

'? Tru tung ve cua (1) cho (2) ta co :

(1) (2)

Do do : y ton tai khi ~ 'y ?: 0 ~ Z2 - 2~ ?: 0

[z ?: 2 ~, Z < 0

Mat khac (3) ~ XZ - Ax + ZZ = b.

V~y x t6n tai khi ~'x ?: 0 <:> -+ - ZZ ?: 0 <:> -2 ~ z < 2 (6).

(5)

Vay h~ co hai nghiern x = -1 hoac x = 2 .

.

! Co the thay ngay khi thay x = -1 vao (1) ta co --l- = D. V*y lam

sao x = -1 13. nghiern cua he ? Can luu y HS ding:

{A = 0 ~ {A = 0

B=O A-8=0

Lei giai tren vi phAffi tinhruongduongvi hieu ding:

//{A = 0 "\ i

" ¢> A - B = 0 \ : /: (.. .,-

t, B = 0" / ... J:>.i'-\!-I;;l Ll./,LV.",

'-.'/ .. ---><:.._ (1 i , , ..

I {A =0 -, .. )1 k.·.--·· "--. lr.. -"

Th\1c ra thi (. B = 0 :::> A - B = O. )C1V, ,-~ ,E::" ~ .-, .... :~

'-.'" \ I' J.' ) "', \ / 1i1/

L'· . .,. d" I.:!.' ~" . _.. • .f'- . I)') " ~~

(It guu ung ta : '\'\~;l './'(')' ,-:! '0.' .. ',

H~ tuong duong voi ' .-/, ,_L (f,,',·'~1

r x' - 3x1 + X + 1 - 0 (1) I

t Xl - X - 2 = 0 (3) .

Ta co (3) ~ [~ = ~ 1

Thu x = -1, x =

2 Can IUQt vao (1) khong thea man nen he dft

. l \,.

cho vo nghiern.

Thi du 2. Giai h~ :

r y + 2. = (3

(2z - y)(y + 2) - 9 + -l-y

l X2 + Z2 = 4x

. z > 0

'? Ta co (2) ¢> yZ + 2y(3 - z) + 9 - 4z = 0

(1) (2) (3) ( 4)

'1

Ket hop (4), (5), (6) ta co z . 0 hoac z = 2.

* r~ uong hop z = 0 thay vao (2) ta co y = -3, thay vao (3) ta co x = 0 hoac x = 4.

'" Truong hop z = 2 thay vao .(2) e6 y = -1, thay vao (3) e6 x = 2. T6m lai : H~ e6 3 nghiern

{~ ~ 3 ; {; - ~ 3 ; {; 2 1 .

z=O z - 0 z = 2

C6 HS thuc hien xang loi giai tren con nghi r~ng : "he ra thua

lphuong trinh (1)" (?)

Bai roan giai he tire Ia phai tim cac gia tri ella fin thea man d6ng thoi tat en cac phuong trinh, bat phuong trinh trong he, Sau khi tu (2), (3), (4) tim ra (xo;Yo;zo) can phai thu vao (1) xern e6 thea man khong.

Ddp so dung la ; (x, y, z) = {(4; -3; 0); (2; -1; 2)}

Thi du 3. Giai va bien luan h~
{ ax - by - J
-
bx + ay - 1
thea a, b.
?Ta co D la -b , ,
- Ib - a- + b~
-. ... a
D 2 -b 2a b
- +
x 1 a
Dy a 2 2b.
- - a -
b 1 ,. 0 ~ [~ : ~ thi

o

x

x =

D

r:

,hUl,I:1

h~ e6 nghiern duy nhat
b Dy a - 2b
y - -
b2 , 0 , + b2
a- 36

ne u a. = b. = a2 = bz = 0 thi tho a man voi mot x, y khi

C I = C2 = ° va vo nghiern khi c, ~ ° hoac Cz ~ 0, luu Y trong truong hop nay khong xet D, Dx, Dy• HS chi bat d~u xet D, Ds, D, khi

,

at + a~ + bt + b~ :;C O.

Kit qua dung ia :

O 0' 2a + b

* a ~ hoac b:;c thi x =, b'

. a- + .

a - 2b y -

, a- + b2

Neu D = 0 ¢> :1 = b == 0 thi Ox == .Ov = 0 nen he co vo so nghiern. ! Day la sai lam kha pho bien cua HS. Co the thay ngay neu '1 = b = 0 thi he vo nzhiern.

... . _.

(

la x Can luu v HS ranz he II

. -" a .. ,x

:~ a = 0 va b - 0 rhi he vo nghiern. Thi du 4. Giai va bien luan h~

{X2, - 2x - 3 < 0 r-x-m>O

(1) (2)

? Tru tung ve ella (1) eho (2) (viP) va (2) nguoc chieu nhau) ta co :

-x + m - 3 < 0 (3)

¢> x>m-3

Chung to voi moi m, he co nghiern x > m - '3. ! C6 the thay ngay : (1) ¢> -1 < x < 3

Vay thi sao tap nghiern eho boi loi giai tren lai co the la x>m-3?

Thu thay m = 2 vao h~ ta co :

{X~ - 2x - 3 < 0 x- - x - 2 > 0

{-I < x < 3

¢> [x > 2

x < -1

¢> 2 < x < 3.

37

Neu nhu theo ket qua "tong quat" cua loi giai tren thi nghiern eual '

, ( /

he phai la x > -1 (!)'. . . 1'\ . j ; c-. ttL-

Sai lam Iii HS dii su d\1Ug tinh chat J r~~~/~ i' : .>:

{:: ~~a-c>b-d /-----

{ea < b >d=>a-e<b-d

cua bat ruing thuc, dung de suy ra bat dang thuc he qua (tat nhien In bat dAng thuc dung neu hai bat dAng thuc 6 gia thiet la dung)

vao viec "bien d6i nrong duong" he bat phuong trinh. Do chat "he

~ .... ' . - ... . . .' ,. -

qua" cua (3) qua manh rna tap nghiem dUQC rna rcng qua Ion va dan den sai lftm. V6i loi giai tren, HS chua thay muc d9 phuc tap eua bai toano

38

Thi du 5. Giai h~

{(X' - 2x - 3)(x;: - -lx + 3) > 0
x- - 4x + ., 2: 0
.J
r 0
'~H~ wong duong voi {~; -+x + 3 2:
2x ., 2: 0
.)
.. F~ > ,.,
.J
1 r .,
s ~-Ix 2: .J
l t~ ...
> .) x s -1.
I..
< -1 Hay xern lai thi du 3, muc 1.2.2. Loi giai tren lam mat nghiern

x - 1.

Ddp sd dung la ; x E (-00; -1] U {l} U [3; +00). Thi du 6. TIm m de h~

{X + Y + xv - 3 xZv + v-x - m

. . .

co nghiern.

Ke'[ qua dung la m s 2 1.2.7. Sai him khi tinh giG; han.

Tiep x'ue voi cac bai toan tinh gioi han. HS buck tu "rnanh dat

\

? Dat u - x + v va v = s:» ta CQ Iu + v = 3 lu.v ~ rn.

Thea dinh li Viete dao thi u, v,la cac nghiern cua phuong trinh :

'.., 0

r- - .Jt + m = .

(*)

H~ co nghiern ¢:> (*) co nghiern ¢:> .G.'t ~ .. 0 9

¢:> 9 - 4m ~ 0 ¢:> m s ~.

! LOi giai tren moi chi dam baa cho su ton tai u va v . Cling thea dinh li Viete thi x, y If} cac nghiern cua phuong trinh

ZZ - uz- + v = O. Do do he hai an x, y co nghiern khi he hai ~n u. v c6 nghiern thea man UZ - 4v ~ 0

t,u + ..,
v - .J
¢:> H~ u.v - m
- 4v > 0 ( 1)

(2) co .nghiern. (3)

huu han" sang "rnanh dfit vo han" voi nhting dai 111Qng vo cung be, vo cung len nen rat de bi mac sai lam. Cac sai lam xuat phat tu viec khong narn vung q uy tac van dung cac dinh li ve gioi han, dac biet la pharn vi co hieu luc cua cac dinh li.

tt« du 1. Tinh :

L = lim ( , 1 1 +

n--;'OO ~ Il" +

1

1

+ n )

r==r=='~2 + '.. + ~n" +

.) To " L .: u 1 [. 1 I' 1

. ia co - 1m .1 1 ·1 + 1m .1 ~ ., + ... + 1m .1 '

n--tooo '4n~+ rT-+OO '4n- + ~ n-+oo '4n- + n

/

= 0 + 0 + ... + 0 = O.

39

1

1

1

{llY voi k -

1, n

! Cnn luu y HS r5ng~ dinh li v~ cac phep roan cua gioi han chi phat bieu eho huu han .\ hang. LOi giai tren da ap dung cho gioi han cua mot t6ng vo han cac so hang nen dan toi sai lam.

Loi giai dung la : Ta e6 :

:5 ~n': + n

Do do : n

~n2 + n

n 1

:::; 2: ~ , :::; 1 .

k=[ n- + k

:Vfa lim n = lim 1

n-TQO ~n'; + n n-TOO N

1 + - n

- 1

81G~/

/--

~--

- 1.

n

Thee nzuven Ii kep thi lim ') . ,1

- • . -..J ,j Fl" + k

k = [ ,

Thi du 2. Cho In = J tg'xdx, Tfnh lim In

!) n-+,.

? Ta co voi n ;::: 3 thi

I)

.T/4

r - f tgn - ~x tg\. dx

:1 _

o

! 1 I--l)dx=' \ cosx

T ~

= J tgn-z x. d(tgx)

u

.T 4

J tg:1-Z xdx

'l

1

_---

n 1

1 Do do In + In-2 - -n--l

Gia ::ill lim In - L thi lim In-2 - L. Suy ra :

n-+oo

0 lim 1 lim (In In - z)
- + _
n--"JO n 1 n-+oo
- lim In + lim i =.L + L - 2L
n-1
n- +,. n-+» HS rang chi co lim (u + v) = lim u + lim vn khi lim u va lim v

..." n n n n n

n-+'»

n- .... ~

.. _,

k ,

.'

Til do ta co L = o.

I Lei ziai thieu viee chunz minh lim I ton tai. :Jhi nho cho

• ~ . ~ n

n-~':G

Jeu phai ton tai hilu han.

Thi du 3. Cho {u } voi Un = (-1)°

Ch ung minh {u } kh6ng co gioi han. ?Ta co Ul = -1 < Uz = 1 > U3 = -1.

Chung to {u } khong tang, khong giam, Theo dinh li Veierstrass thi {u } khong co gioi han.

! Sai 11m cua loi giai do khong hieu dinh li Veierstrass, dinh li

co gioi han.

Lui giui dung fa :

Gia su lim u, - L thi max {II - LI ; 11 + L i} ~

n--JCI

il LI + 11 + L' 11 L + 1 + LI
I -
~ ~ , - 1
2 )
-
\
Do do, voi e 1 h' i 1 - LI > e hoac 11 +LI e tuc la
- - t 1 >
! I U~k - L I > c hoac i U::k-1 - L I »e VOl \ik E N°. Chung to mau thuan voi gia thiet {u}. - L, nen {u} khong co gioi han.

I 3x + ') ') :x tu du 4. Tinh lim \ ~ -'-1

~_ " .JX

41

khu

dancr . ~

-,

nhung

sai

lftm

xuat . hien

khi

viet

-

~.:: ...

~, -,

~x': + I Thi du 5. Tinh lim ----

x + 1

x-:c

~xZ + 1

? Ta eo lim = lim - _

x-oo x + 1 x-oo 1 + 1

- 1.

x

! LOi giai tren dol chia ca tu va mau cua nhan thuc ~ x, + 11 cho x +

Dieu do chi viet dUQC khi x > o.

r x-I _.

i 1 VOl X

Thi du 6. Cho f(x) = ~x _ .

2 vdi x s 1

> 1

x-I ~x - 1 . ~x - 1

"Ta co lim reX) = lim = lim __ --;:::====;:=--_

:<-1 x-I ~x - 1"(_1 ~x - 1

Tinh tim f(x).

:<-1

= lim ,jx - 1 = O .

. x+-t

~ Lei giai tren da mac nhien chi xet x > 1 nrc lit xet gioi han phai khi x tien tei 1.

Lai giai dung la : .

lim f(x) = lim _x;==:==l;=

~ + ~x - 1

x-I .'<-1

'Jx - 1 ~x - 1
- tim ~x -
+ 1
x-I
- lim ~x - 1 - 0
+
x-I 42

lim f(x) - lim 2 - 2.

\In ~{i1 {i1 , s {Q. 2) + 26
-
n
Do 1 ii1 1 :2 :2 va lim (1
< -s - + ,--
n ~n . O--"D
suy fa lim f\[il - 1 tuc la lim v - 1.
n
n-" ..I.,,» n .......... :xJ 1

1 - +

n

VI lim f(x) ;t= lim f(x) nen lim f(x) khong ton tai.

x-I

Thi du 7. Chung minh voi rnoi so tu nhien n ~ :2 thi

2

') Xet {u } voi u =? - 1. ta co lim u - 2

. n n.... n n-+ao n

Xet {v } voi v = "'In

n n

Do n ;::: 2 nen n{i1 > nIT = 1 t

Mat khac, ap dung bat clling thuc Cauchy cho n - 2 so bang 1 va :2 so la {i1 thi

'1 ')

T ;._ 'j L

n '.In .'

Vi lim u - :2 > 1 = lim v :::> U ;::: V VOl n ;::: 2.

n n n n

n-+~

' Ta co dinh li : "Neu {u }. {v) co gioi han va Un ~ v; thi

lim un ;::: lim v,,". Nhung die u nguoc lai, ket luan ur1 ~ "«

n-""'lQ n-"T'OO

\:I n > 2 nhu loi giai tren Ia sai. Menh d~ nguoc lai co the phat bieu :

"Neu {un} va {vn} thea man lim u, 2: lim Vn thi ton tai n' sao cho

n- +00 n-""oo

\:In ;::: n' thi u, 2! Vn". Dieu quan trong la doi voi bai roan cu the nay, tai sao n· co the chon If! 2 ? Khang dinh chon rr = 2, chinh If! giai thi du 7.

43

Lai giai, dung la :

Ap dung bat dang thuc Cauchy eho n so duong gem 1 so la n va n - 1 so la 1 ta co :

n + (n - 1) > r\[il n

1

~ 2--2: rfi1

n

Dang thuc khong xay ra vi n 2: 2.

1.2.8. Sai him khi giai cac hili toan lien quan to; dao ham

Cac sai lftm lien quan toi khai niem dao ham thuong gap khi tinh <;l~Of ham va khi van dung dao ham de giai toan.

>." "

Thi du 1 :

{sin:: x

khi x ;:: 0

Cho f(x) = Xo

khi x = 0

Tinh r(O).

?Ta co f(O) = 0 ~ f'(O) = (0)' = O.

! Loi giai da mac sai lam Ion khi thay x = 0 VaG f(x) r6i moi tinh .

--.-.-.--~---- ,

f(O)

1° f(O + ~x)

= 1m

.l':<-I) ~ X

_ [lim s in ~ X];: = 1.

u:<-o ~ X

Thi du 2. Tinh dao ham cua ham y = (x' + 1):< ? Ta co y' = x (xZ + 1) x - 1 • (x2 + 1)'

44

( .:! + 1).'< - i ,

= X X .' .:..X

= 2x':! (xz + 1 r - 1

~ Lei giai da van dlfng (x")' = n.x" - 1

Van dung nhu v~y la sai, vi 0 bai nay y = (XZ + 1)" thi mtl x la bien so chu khong phai la httngso nhu 0 cong thuc tren.

Thi du 3. Tinh gia tri cua

1 f(x) = arctg x + arctg-.

x

'.

voi x ;c O.

'? Ta co f'(x) -

1

1 1 0
- , 1 -
X" + 1 X" + 1
Chung to f(x) - C voi x ;c O.
Ta c6 f(1) arctg 1 arctg 1 it .1: :r
- + - + - -
4 + "
.:..
,-,; '(x st O.
nen c - 2 => t ,x) - :;- v6i moi x ~ ! Co the thay ngay f( - I) = aretg( -1) + arctg( -1)

= (- .~) + (- .~) = - J; .. V~y sai lftm 0 dau '~

Ta biet ding "Neu f(x) = c voi x E (a;b) thi f'(x) = 0 \:Ix E (a;b)", Dieu nguoc lai, "Neu rex) = 0 'rfx E (a.b) thi f(x) = c \:Ix E (a.b)", Lei giai tren da coi R \ {O} la mien c6 dang (a.b) de van dung dieu nguoc

lai do ia sai lam. '0 day can van dung menh d~ d6 cho tung khoang (-00 ; 0) va (0 ; +00).

Kit qua dung La : f(x) =

tt ,,'" 0

- - neu x <

2

It

neu x > 0 2

45

v -

..

/. ,,'

II., I

i .'" ( CU. i. 1,(0.(,". _.

Thi du -I. Tim m de ham so

':! = t x3 - mx- + (m + .2)~ + 1 d6ng bien treri R.

'? Y dong bien tren R ~ i > 0 \:Ix E R.

¢> XZ - 2mx + m + '2 > 0 \:Ix E R ¢> ~' < 0

' :<

¢> rn- - m - :2 < 0 ¢> -1 < m <: 2.

! y' > 0 voi x E (a;b) la dieu kien du d~ Y d6ng bi€n tren (a;b), chu . khong phai la di~u kien can. Chang han y = x' &mg bien tren R, nhung y' = 3xz = 0 khi x = O. Nh6 rang, neu y = f(x) xac dinh tren (a.b), f'(x) ~ 0 \:Ix E (a:b) nhung f'(x) chi trier tieu taihiluhan diemthuoc (a.b) thi y = f(x)

'~ . '" , It· { . (i {/ .e ' I .: A--J --( ,

dong bien tren (a;b). :. l'.'! ('" }" 1'- \.- t.: i' V(i ;//C:" {, .

_" I' I

Lili giai dung La : ( c ~ (:';11' (-."/ C:ln/ ,P(J. Lf: (;)1;) r

:'1

y dong bien tren R ¢> '1' ;:: 0 T:/x E R ¢> x;: - 2fux + m + 2 2: 0

\:Ix E R ¢>~' ~ () ¢> mZ - m - '2 < 0 ¢> -1 ~

x

Thi du 5. Tim tap gia tri cua ham so

x + 3

? T~p xac dinh la R

Ta co

v' =

-3x + 1 (x2 + 1)31Z

Do d6 bang bien thien cua y la :

x

- :0

1

..,

.)

v'

+

v

.+6

47

Suy ra tap gia tri cua y la (- 00: -no].

! Sai Iflm la HS tu6ng lim f(x) = - 00.

Nhung lim f(x)

-1 va lim f(x) == 1. Do do tap gia tri cua y In

(-I; {WJ.

1.2.9. Sai lam khi xet bai toan ve tiep xuc va tiep tuyen

Cac sai lam khi xet nhung bai toan loai nay xuat phat til viec khong narn viing thuat ngt;i hoac kh6ng hieu'dung su tiep xuc cua hai do thi ta gi,

Thi du 1. Cho ham s6

y == x3 - 3x + 1

Viet phuong trinh tiep tuyen ke tv diem A(3; 19) toi do thi.

: Ta thay f(3) = I q => A thuoc do thi. Vay phuong trinh tiep

tuyen can xacdinh ta

--I ~_ I: .:

i. .. I \' ,'! ;

I !J.:.~ f • \..V.i" I ', ',. L' __ I... ':',

--(: '. :_/ /1,

(L{:. ,-',_ -. it' 1.,1 ~,- f_A / tIl ..

, . " , r '_,

¢:> v = 24x - 53. '/j 'T(' .' . l . , ,Ij: ,

. JIF,. ,-,'Ct." i'Lle') 1"~1 ~

. ! Phuong trinh tiep tuyen y = 24x - 53 lit tiep tuyen tai A (nhan /(L:

A lam tiep diem) ,tat nhien la ke til A. Nhung van co the co tiep tuyen ~~Iji diqua A mit A kh6ng phai Iii tiep diem. L)L-

y - f(3) == f'(3)(x - 3)

Ket qua dung fa :

Co 2 tiep tuyen tho a man bai roan :

y = 24(x - 3) + 19

y == ~ (x - 3) + 19.

Thi du 2. Tim cac diem tren true Ox rna tu do ke duoc dung mot tiep ruyen to. do thi y = Xl - :

x-

48

t' >-

: G9i diem can tim tren Ox 121 :V1(~:O). Duong thang d qua M co dang . = k(x - h).

Duong thang nay la tiep tuyen ella do thi khi phuong trinh

x:! - t = k(x - h) co nghiern kep

x-

~ x:! - 4 = k(x - l)(x - h) co nghiern kep

~ (k - 1 )X2 - k(h + l)x + kh + 4 = 0 co nghiem kep ~ ~ = 0 ~ k:!(h + 1)2 ..... 4(k - l)(kh + 4) = 0

x

: ~ k~(h - 1)~ + 4(h - 4)k + 16 = 0 (*) -

De co mot tiep tuyen thi phuong trinh (*) co dung mot nghiern . ~ ~. = 0

k

4(h - 4)2 - 16(h - 1)2 = a

[h ~ 2

~ h = - 2.

: Vay co hai diem thuoc Ox thoa man bai roan la ivI,(2;O) va ivL( - 2:U).

! Lei giai tren lien tiep mac sai lam

• Khu x - 1 6 m~u khong co dieu kien.

• Phuong trinh an x co nghiern kep con thieu dieu kien

k - :;c o.

• Phuong trinh (*) co 1 nghiem con thieu yeu cau k;t: 1 nen d~n to: thieu cac truong hop can xet.

De giai dung bai to~n nay phai narn viing phep bie'n d6i tuong duong va bien luan du cac truong hop xay ra.

Thi du 3. Viet phuong .trinh tiep tuyen cua do thi y = x2 - X + 1,

biet ding goc giila tiep tuyen voi duong thang y = 0 la 45°.

la aI' a: va g6e giua cluing la a thi tga =

a - a

1 2

'? Gee giua tiep tuyen va duong thang y = 0 la 450 khi h¢ so goc

. .

cua tiep tuyen la tg45° = 1.

Do d6 tiep tuyen co dang y =' x + k. Duong thang nay In tiep tuyen ella do thi ~ phuong trinh XZ - x + 1 = x + k co nghiern kep ¢> x': - 2x + 1 - k = 0 eo nghiern kep ¢> il~ = 0 ~ k = o.

Suy ra e6 1 tiep tuyen thea man bai roan la y = x.

! Lei giai tren del kh6ng chu y toi khai niern v~ g6e giila hai duong thang, Can luu y HS ding : neu h¢ so g6e ella hai duong thang Ian luot

o day duong thang y -

o co he so 0'6e a - 0 nen he so zoe a

, .::> 1 .::-, 2

cua tiep tuyen thea man I a.] = 1 ¢> az = ± l.

Lei giai tren thieu truong hop a., = -l. Ke t q uil dung til : y = x va y = - x + 1.

Thi du 4. Tim cac diem cua true tung rna tu d6 ke duoc dung ba tiep tuyen toi do thi

y = x ' - x2 + 1.

? Do thi co true doi xungla Oy. Do d6 neu d la tiep tuyen ella do i thi khong vuong goc voi Oy thi d' doi xung voi d qua Oy cung [a tiep tuyen cua do thi. Suy ra tll 1 diem tren Oy ke duoc dung ba tiep tuyen toi do thi thi trang die tiep tuyen d6 phai c6 tiep tuyen vucng g6e Oy. GQi tiep tuyen nay la y = m thi he

(X"~ - XZ + 1 = l4x:3 - 2x= 0

lr-xx - 0 7

Taco (2) <:> _ + j_.

- 2

m (1) , hi

(2) co ng iern

49

Neu thay x == 0 vao (1) ta c6 m L~li A(O;l) nen A(O;l) thoa man.

1 =:> tiep tuyen y == 1 eat Oy

~, ~ 3

N " h ~ .: , (1) , .J .• ' " ,

L eu t ay x == ± 2, vao ta co m == 4 =:> tiep tuyen y == 4- cat

~ ~

Oy tai B (0; ~) nen B (0; ~) thoa man.

T6m lq_i, A(O; 1) va B (0; !) .'

! Ly luan phan d~u cua lei giai tren loa dung, nhung viec suy ra trong cac tiep tuyen phai c6 tiep tuyen vuong g6e voi Oy thi day chi Iii

dieu kien can. Khi tim duoc m == 1 hoac m == ! de c6 A(O; 1) hoac

~

B (0: ~) lei giai tren VQi Yang kh.1ng dinh ca 2 diem nay thea man nen

del sai lam. Can phai thu lai se thay voi A(O; 1) co 3 tiep tuyen di qua,

_.,

con voi B (0; ~) ,thi c6 dung 1 tiep tuyen di qua.

'? Gia su A(O ; m) la diem cua true tung. GQi Xo la hoanh dQ tiep diem cua tiep tuyen di qua A thi m == f' (xo)(O :- xo) + f(x,)

¢:> 3x4 - XZ + m - 1 == 0 (:,c)

0.0, .

I

Tu A ke duoc 3 tiep tuyen ¢:> phuong trinh (") c6 dung 3 nghiern

khac nhau.

Di~u kien can de (*) co 3 nghiern la Xu trinh (:!:) ::;> m == 1.

Th lr lai : voi m = 1 thi (*) tro thanh 3x~

o thoa man phuong

~ X" • 0

o

== 0

== ±

3

V*y A(O ; 1) thea man.

50

~ X2 + X + 1 - k = 0 vo nzhiem

~ .

'\ ..

!

.:. I, .. r ,

? Sau khi 11 lu~n (*) c6 3 nghiem phan biet, dat t = den phuong trinhg(t)

Vay A(O ; 1) thoa man.

! Hai ldi giai sai l~m d dau ? (Mac dii co dap so dung l). C~n luu

y HS dmg : si tiep s!i§tp. C?~_~~~~ __ ~9_~Q __ ~~~~gduI1gb~ng ~o ti¢'R_tllY~Pl __ nhat la doi voi ham. so b?c-+. V oi 3 tiep diem, bier dau ch i co 2 tiep

tuyen vi co nhting tiep tuyen c6 2 tiep diem voi d6 thi. I~i~ao~~.6I1g xet phuong t~il1~_(~J~6_~ __ I1_g_~j_~g1_P_l!~_IJ._.!?i~!_vi biet dau voi 4 tiep diem

ay lai co dung 3 tiep tuyen di qua A ?

•• • - ••• _'< •. _-- - _-"--.- '- •• - -_ •. -._---- •. _-_-_.' -

Thi du 5. Tim III de d6 thi y = x3 - k(x - 1) - 1 tiep xiic voi true hoanh.

? Do thi tiep xuc voi true hoanh .~ phuong trinh . I -: tt .

x' - k(x - 1) - 1 = Ot6 nghi~lTl' duy nh-a:~-·)l.l /C. i. >,{,:; ;<, _i.

. ._ ... _ ..... _---_._---- '----_'-"-- .. _.. '-"-.

~ (x - 1) (x- + x + 1 - k)= 0 c6 nghiern duy ~hf(t--

= 3t2 - t + m - 1 = 0 c6 nghiern t1 = 0 va t;: > 0

¢ If > ~ ~ 0 ¢ m ~ I

. {a(o) = 0

<=> 0 .

S > 0

. ~

;) ~ ~ < 0 ~ -lk - 3 < 0 ~ k < -.

:< -+

--,i,.r ..»,

..

! Do truoc day, nhieu HS quen xet viec tiep xtic cua dUdng bac hai va duong bac nhat hoac cua hai dUdng bac hai se thay hinh nhu (")

khi hai dUdng c6 mot diem chung thi se tiep xiic nhau. Su suy dien ra thanh mot tinh chat tong quat cho hai dUdng bat ky tiep xuc nhau

khi va chi khi chung co mot diem chung la sai lam .

Lui giai dung to. :

D6 thi tiep xiic Ox <=> phuong trinh xJ - k(x - 1) - 1 _ 0 ::6 nghiem kep , :

51

:.1 - () ¢> -+k - 3 = 0 ¢:> k

.'( !

-r

¢> (x - 1) (X2 +x + 1 - k ) = 0 co nghiern kep,

-.

!: Trirb ng lu!p 1: t(x) = XZ + x + 1 - :( co nghiern kep ~

r( '" Tru'iJn~ t(x) co nghj~m x = 1 ~ t(l) = 0 ~ 3 - k = '0 ¢:>

--.-- .... -.-- , , ! ~ I I ) ') () f //

k = 3. \ .i '/ '-. i' i ~ .. j ~tV«(_.·tj). _/--

\ ,- ,1- L).1. v·",-,I_,c '1 /"

" -l_.\' I I

<: '4. 3:1

_r Tom lai : Do thi tiep xuc voi Ox ¢:> k = - hoac k = 3. 4 .

Chit j : Mot so HS khi giai nhu tren lai sai lam khi yeu Call 6 truong hop 1 ta ttx) c6 nghiern kep khac 1. Neu t(x) c6 nghiem kep x = 1 'thi van thoa man bai toano

1.2.10. Sai lam khi xet cac dirong tiern c~n cua dc3 thi,

Khai niern ve duong tiern can cua d6 thi lien quan chat che toi phep tinh gioi han (ke ca phep tinh gioi han mot phia), Nhieu HS khong nam dUQC dinh nghia rna chi nhin vao hinh thuc ella ham so va

suy dean mot each may moe nen dan toi sai lam. Tat nhien viec tinh cac gioi han sai cung dan den sai lam khi tim cac duong tiern can .

. .,

Thi du 1. urn cac duonz tiern cdn cua duonz v = .J

: ---.. vJ • ~ 1 - X"

..

.) VI lim v = :0 n e n d6 t h i co h a i ducrig tiern can dung lo1

.'{-:=!

X - + 1.

Vi tap xac dinh cua ham sola (-1; 1) nen lim y khong ton tai, suy

!C-

ra do thi khong co tiern can ngang.

! VI tap xac dinh la (-1; 1) nen chi co lim y - + co va

+ x--l

lim v

- +00. Do do kh6ng viet lim y _ = 00

~-!:I

x-I

v -

x2 + x - 2 X - m

Can luu v them d6 thi cuns khonz c6 tiern ca. n xien vi tao p X3C

J .::;,::;,

Liinh cua ham so la (-1 ; 1).

Thi du 2. Tim m de d6 thi

khong c6 tiern can dung.

'? V oi rnoi m thi lim y = 00 nen d6 thi luon c6 duong tiern can

x .... m

,

dung x = m. V~y kh6ng c6 m thea man bai toano

! Sai 111m khi viet lim y - oo , Nhieu HS khi tim duong tiern can

x +- m

dung ella d6 thi ham phan thuc y = ~f:i thuong giai phuong trinh v(x) = 0 de tim xC?) va nr do suy ra ngay (?) tiern can dung. Day la su may moe khong c6 co so nen nit de mac sai lam.

Thi du 4. Tim tiern can cua do thi xJ

y =

X + 1 .

? Ta c6 lim y = oo nen d6 thi c6 tiem can dung x = -1.

x--I

Ngoai ra lim [y

(ax + b)j -

I, xJ - (x + l)(ax + b) " xJ - ax:! - (a + b)x - b

- 1m - 1m

x- '" X + 1 X-'JO X + 1

nen d6 chi kh6ng e6 tiern can ngang va tiern can xien,

l Luu y: limy == -00 va limy'= +00 nen d6 thi cc.tiem can

, ~

x--l

. dung x = - 1.

Dung la d6 thj khong co tiern can ngang va xien, nhung neu luu y :

1

y - XZ - x + 1 - --

x + 1

54

chi lim [y - (x2 - X + l)]

o nen d6 thj co duong tiern can la parabol

V x.:! - '( + 1.

1.2.11. Sal Him khi lam toan giai tich t6 hQ'P

Day la 109i roan moi duoc dua vao chuong trinh roan 6 pho thong cho nen nhieu HS lung tung va dan toi sai 15m.

Co so de giai cac bai roan la viec van dung cac quy tac nhan, guy rae ccng va cac khai niern chinh hop, hoan vi, teS hop. Cac sai lam do kh6ng biet la van dung guy uk nao, khai niern nao cho bai toan dang

lfl" .! 1" ~ .::> d.

Thi du 1. Mot da tiec co 10 nam va 6 mi deu khieu vii gioi. Nguoi ta chon 3 nam va 3 mi de ghep thanh 3 cap nhay. Hoi co bao nhieu each ghep 3 cap nhay ?

.~ ;\;loi each sap thu tu 3 ban nam trong 10 ban nam la mot ch inh hop 3 cua 10 ne n so each chon 3 ban nam co thu tu la

A~u = 8.9.10 = 720 each.

Tuong tu so each chon 3 ban nii co thu tu la A~ = 4.5.6 = 120 each. Vay so each be tri 3 cap nhay Ia

A~o· A:' = 720 x 120 = 86400.

! T9i sao lai sap thu tu ca 3 ban nam va 3 ban mi. Gia S1.1 co 3" ban nam thea thu tu la A, B, C ghep nhay voi 3 ban ml thea thu t\l la a, b, c rue la ta co 3 cap nhay (A, a), (B, b) va \C, c). Neu lay thu tu khac cua 3 ban nam la (A, C, B) va thu tu khac cua 3 ban mi Ia (a, C, b) thi ghep 3 cQ_P nhay (A, a), (C, c) va (B, b) van la each ghep 3 cap nhay truce. Sai lam dlm toi so each ghep lon hen thucte vi co nhting each ghep 3 C?P nhay duoc tinh nhieu ian. Ket qua dung la. 14400 each ghep.

55

'\

Tlii tilt 2. Tel 5 chti 50 O. 1, 2, .3, -+ co the viet duoc bao nhieu so '0 -+ chu so khac nhau ma:)o nav nhai co ch u sol ?

I".. ... j ,

chQn. Nloi each chon a:c, <13, 3-1 la rnot chinh hop chap 3 cua 4. chit s() con lai, nen voi moi each chon a~ se co A~ each chon a2, a3' a.j',

Do d6 so cac so thea man bai toan la

4. x A~ = 4. ,< 4,! = 96.

! U luan tren quen milt (va chua co gi bao dam) de so thoa man bai roan co ch u so 1.

Chu ~v : C6 the xet a1 1 thi rnoi each viet a2, a3, a.j la chinh hop

chap 3 ella -I- chii so con lai nen co A~ so. V oi a:: = 1 thi a1 co 3 each

viet va khi do rnoi each viet a3, a.j Ia chinh hop chap 2 ella 3 chilsc con lai. ne n co 3 x A~ = 18 so .

• , .J

Tuong tu, a3 • 1 c6 i8 so, a-l = 1 co 18 so. V $.y so cac so thea

man bai roan la A~ + 3 x 18 = 78 so.

Thi du 4. Trang mot buo: hQP mat co 5 ban narn va 5 ban mi. C6

baa nhieu each xep cho ngoi quanh mot ban tron sao cho cac ban narn, rni ng6i xen ke nhau ?

? Hay co dinh cho ng6i cho 1 ban nam thi moi each xep 4 ban nam con lai la mot hoanvi cua -+ nen e6 4! each xep. Mat khac 5 cho ngoi cua 5 ban mi la xac dinh nen c6 5! each xep cac ban ml.

V~y sO' each xep cho ngoi cho 10 ban tho a man bai toan lit -I-!' 5! = 2.+ x 120, 2880 each,

! Doi voi bai toan xep eh6 ng6i cho mot cuoc gap mat xung quanh mot ban tron dang co nhieu quan diem. Nhu the nao la mot each bo

56

rri vi crt ngoi cho rnoi nguo: ? De bai khong noi r6 thi hieu nhu the nqo day'? Ta chi quan tam 'Ji thu tu ella moi nguoi voi nhau hay quan ram t,Oi ca vi tri ngoi? V oi mot thu ru da dinh van co the bo iri vi tri

• • ".1 •

nlJoi khac nhau.

~ -

Chang han co 2 ban thai thi chi co mot thu tu bo tri 2 ban, nhung vi tri.rigoi thi r6 rang la co hai each bo tri vi tri ng6i khac nhau.rNeu

~ -" : I i

gia t~iet bai toan kh6ng noi r6 nhu the nao Ia hai each bo tri ngoi khac

nhau.thi loi gia: khong xac dinh hoac If! cac b~n, phai bien luan.

Neu hieu nhu binh thuong thi tiiy thu tu ng6i khong d6i nhung vitri ngoi khac nhau la cac each be tri, khac nhau thi loi giai tren Ia

sai Iftm. Khi do loi giai dung la :

Hay bo tri cho ngoi cho ban nam Al thi co 10 vi tri khac nhau. V oi moi vi tri cua Al thi moi each bo tri 4, ban narn con lai If! mot

.'

hoan vi cua 4 nen co 4! each xep cac ban nam va moi each xep vi tri cho 5. ban mi la mot hoan vi cua 5 nen co 5! each xep.

V~y voi moi vi tri cua ban nam Al se co 4! 51 each be tri cho ngoi cho 9 ban con lai.

Do do co 10 x 4! 5! = 28800 each bo tri thoa man bai tcan. Thi du .4. Cho 10 diem tren mat phang trongd6 khong c6 3 diem nao thang hang. Xet tap hop cac duong thang di qua 2 diem cua 10 diem da cho. Hay tinh so giao diem khac 10 diem da cho, do cac duong theing nay tao thanh nhieu nhat Ia bao nhieu ?

.? Moi duong thang di qua dung 2 trong 10 diem da cho (vi kh6ng . co 3 diem nao th5ng hang). Do d6 so cac duong thang co duoc la

C·'

in =

10! 8! 2!

,- 45.

1_

De s6 gino diem tao thanh do cac duong thttng khac 10 diem da

.

cho la nhieu nhat thi cac duong thang nay khong eo cap dtr-.ng nao

song song va khong co 3 duong nao dong quy.

Khi do ell 2 duong thang cho Lgiao diem. Nen so giao diem t90 thanh la C;5 ' trong do eo 10 diem da cho nen so giao diem nhieu

nhat khac 10 diem eta cho la C~5 - 10 =

4-'

43~2! - 10 = 980

! V6i m6i diem da cho, khi noi voi 9 diem con lai se c6 9 duong thang va 9 duong thang nay dong quy. Do d6 gia thiet cac duong thang khong c6 3 duong nao dong quy la kh6ng the co va tv day dfin t6i sai lam ella 16i giai. Ket qua clung If! 630 diem.

Thi du 5. Cho -l- diem trong do khong co 3 diem nao thang: hang.

Cu qua 2 diem trong -l- diem ngu(li ta ke 1 duong thang, Hay cho bier so giao diem tao thanh khac -ldiem cia cho se it nhat la bao- nhieu ? 4'

'?Thay ngayso cac duong thitng In c~ = 2! 2! = 6.

De so giao diem tao them ngoai 4 diem da cho If! It nhat thi trong 6 duong thang nay phai co nhieu cap duong thang song song nhat. Do chi co -l- diem nen kh6ng the co 3 duang thang song song v6i nhau. VI vay cac cap duong thang song song se khac phuong nhau. SUY ra 30 cap nhieu nhat In 2, khi 4 diem IS. cac dinh cua mot hinh binh hanh . ABCD. Do do so giao diem duoc tao them it nhat la L chinh 1a giao cua AC va BD.

! LOi giai tren se dung neu gia thiet 4 diem da cho thuoc mot mat !

phang. Trong khcng gian hay U1Y -+ diem la 4- dinh ella mot tu dien thi so giao diem t~o~hem ngoai .+ diem da eho se la ... O!

57

Thi du 8. Giai phuong trinh

A3 + 0-2 14x

x .X

Thi du 6. V6i cac chu s6 0. I, 2, 3, 4, 5 co the viet duoc bao nhieu sO gem 8 chii so rna trong do chii sol' co mat 3 l§.n va moi chii so khac co mat dung 1 l'an.

? Xet tap hop A = {O ; 1 ; 1 ; 1 ; 2 ; 3 ; 4 ; 5}

GQi so thoa man bai roan la ala2a3a"aSa6a7aS thi al co 5 each viet - I - ~'l ? .... ,I -

trong .) e ru so ,_,.J, +, .).

V oi moi each viet a] thi moi each viet tiep a:;, a~, a4, as' a6, a., ag If:! mot hoan vi cua 7 phan tU con lai trong tap A, nen co 7! each viet.

Suy ra, so cac so thea man bai toan 18. 5 x 7! = 25200 so, neu nhu coi 3 chii so 1 khac nhau. V6i 3 vi tri nao do cua 3 chu so 1 se cho 3! hoan vi nhu nhau. Do do so cac so thoa man bai roan dting in

.) x 7!

= 4200 so.

~I .J,

~ N eu coi 3 chii so 1 la khac nhau thi a] phai co 7 each viet. Sau

. 7 x 7!

do Ii luan wong tv nhu tren se co dap so citing la "., I :.=: 5880 so . .J,

Thi du 7. Mot cua hang eo 4. ella ra, vao. Hoi eo bao nhieu each vao mot cua va khi ra cua khac.

? ;\loi each chon vao mot eva va ra mot eva khac in mot (0 hop

h ' ~ . f 0 de " , h he - [' C" ..j.! 6' h

c ap ~ eua -to 0 0 so eac t oa man a ; = 2! 2! = eae.

"

! Vi phan biet cua vao va ella' ra nen ffioi each vao mot eva, ra cua khac khong phai 18. mot t6 hop chap 2 cua 4 rna phai la mot chinh 4!

hop chap 2 cua 4. Do do so each thoa man la A: - - 12.

.. 2!

")
- 5
)
-
:2 14x

x!

"Ta co +

(x - 3)! 2! (x - 2)!

x(x - 1) ¢:>. x(x - I )(x .;_ 2) +

x!

::z: l4x

5 Phuong trinh c6 hai nghiem x = 5 ; x - :2'

. ! Lei giai tren chua ccdieu kien de A~ va ~ - 2 c6 nghia. Cfln

phai b6 sung dieu kien nay la x ~ 3 va x E Z. Tv d6, phuong trinh co nghiern duy nhat x = 5.

1.2.12. Sui Him khi giai toan nguye n ham, tich phan

Nhung sai lam thuoc pnan nay lien quan toi su hieu biet khong

dung cac khai niern va van dung sai cac dinh li, q uy taco

Thi du 1. Chung minh : F(x) = -( 1 + xje: Itt mot nguyen ham ella f(x) = xe ". Tv do hay tim nguyen ham cua g(x) - (x - l)e-x.

?Ta co

F'(x) = -e-X + (1 + x).e " = xe " = f(x) ~ F(x) la mot nguyen ham ella f(x).

Ta co : J g(x) dx

J (x - 1) e-X dx

- f x e --x dx - f e -x dx

[-(1 + x)e" + c] - [,- e-X + s] -.

- (1 + x)e-X + e-'< = - x. e:< .

! Sai Cam cua 10i giai tren tuong tu nhu sai lam khi giai(h':~ phuong trinh luong giac rna HS viet chung ki. hieu k voi k E Z eho bieu thuc nghiern ella hai phuong trinh nen khi tru tung ve ella hai phuong trinh da lam mat nghiern cua he.

• 1.,)

59'

I)

J (X + 1)2 dx

1

1 J {U du = 0 2

Dei voi viec lay nguyen ham cling vay, HS hay viet hting s6 C cho

.

mo: phep tinh nguyen ham nen d~n te. sai lftm. Cftn sua lai dO~lI1 cuoi

cua lo: giai tren.

Thi du 2. Tinh J (2x + 1)3 dx .

(2x + 1)4

+ C.

4

! Loi giai tren da van dung cong thuc :

xn+ 1·

Jxndx = ' 1 + c voi n .;t: -l. n + .

o day phai dat u = 2x + 1 =:> du = 2dx :::> dx = d; de co loi giai dung.

o

ttu du 3. Tinh f (x + 1)2 dx.

-,

? Da t u = (x + 1 r =:> d u = 2 (x + 1) dx =:>

. du du

=:> (L\ = 2 (x + 1) = 2 {U . V oi x = - 2 th I Ll = 1 ; x = 0 thi u = 1.

Do do

-'

~ Luu Y u = (x + 1)2 khong phai ham sO' don dieu rren [-2 ; 0] nen ~k116ng [he d6i bien, d6i can nhu loi giai tren duoc. Neu muon d6i bien thi phai viet rich phan can tinh thanh tong cua hai tich phan rna u = (x + 1 f

don dieu. LOi giai tren COil sai lam khi viet dx = J( du 1) - ~

- X + - 2{U ,

nhu vay da ttl u = (x + 1 )2suy ra x + 1 = {U , dieu nay chi duoc "iet khi x ~ -1.

,

'd . T' h f- dx

TIll u u, m -,

. . _I x-

60

,) Ap dung dinh li Newton - Leibnitz :

2 jx;: X-I ';: -1

f -, == J x ? dx = - I = - - 1 -

v» -1 -I ?

-I'\' -I _

,

-.)

!

-

! Ham so y = :z gian dean tai x = 0 E [-1 ; 2] nen khong su dung duoc dinh li Newton - Leibnitz de tinh rich ph an nhu 6 tren. Nho ding gia thiet cua dinh li Newton - Leibnitz la y = f(x) lien tuc tren [a.b], d6 chinh la pham vi duoc su dung dinh li nay.

Lui giai dung La :

V, h' ~- 1 khc ic di h tai 0 [1 ?] ,

I am so y = Xl r ong xac am tai x = E - ; _ ncn tich

phan nay kh6ng t5n tai.

Chu j : C6 the nhan xet lim -\- = + 00 nen ham so kh6ng bi chan

x-v xa:.. . '_. .

tren [-1 ; :J, do do rich phan nay kh6ng ton tai, tnt du 5. Tinh J 1 ,dx.

u -r- smx

? Oat t x h' - 2dt
== to' -;- t I dx -
::;, 1 + t2 "
w
Do do I dx " J dt ? f (t 1) -2 d(t 1)
- = + +
1 + sinx (t + 1 )2 -1

= ') + e -

- . (t + 1)

X

to' - + 1

°2

+ c.

Ap JI,lDg dinh Ii Newton - Leibnitz thi

:r dx _ 2 :r

[ 1 + sinx - x, .)

to' - + 1 °2

:1

tg 2 + 1

+ tgO + 1

2

- ------

61

I} dx

- sirrrz x cosa '

v. -1 I. -, \ - + 1

\ sin« /

1 I d ( x ~i~~sa )

, f---

sine -I (X -, cosa) ~ + 1

, sma I

VI tg J~ kh6ng xac dinh nen tich phan can tinh kh6ng xac dinh,

- ,

~ Day la sai (am cua nhieu hoc sinh hay dung cong thuc IUQng

, giac de bieu dien sinx, cosx, tgx, cotgx qua tg; ,.

Viec tg .I; khong xac dinh 6 tren chi suy ra duoc tich phan da cho kh6ng tinh duoc bang phuong phap do.

Thi du 6. V6i 0 < a < st , tinh

1

f dx

-1 Xl - 2xcosa + 1 .

?Ta co I

1 dx

£ (x - cosar + sin2a

1

, x - cosa

arctg ( , )

SlDa

sma

1 r I 1 - cosa , I - 1 - cosa, ]

arcts ( , ) - arctz ( , ')

smc : L. ~, sma ~ \ sma

~ Lei giai chua the dung lai 0 ktft qua tren, rue la neu dung lai nhu the HS dang con "no" bai to an rut gen. Can tiep tuc loi giai : .

1 ' a a

I = sina [arctg (tg 2) + arctg (cotg 2 ) ]

1 [ a ( tg (; ~))J
--- arctg ( tg 2 ) + arctg -
sma
,a it a (0; ;)
VI-' - - E
2 ' 2 2
o < A. < 1"C nen I 1 (a .1: ~) n
do - + - - -----
sine \ 2 2 2sina 62

-2~2 cos

(X :r),

-+-

,2 -+

3..T

1-;- - IX :r).

0- + 2,i2 cos I - + -

~ \ 2 4

.:!..T

Tlzi du 7. Tinh J ~ 1 + sinx dx.

u

1T 2..,

?Ta co [ ~ 1 + sinx dx = [~ ( sin ~ + cos;) 2 dx

~ ~

f(' x X) "'f( X x . d(_~)

sin 2 + cos 2 dx = 1. 0 sin 2 + cos 2 ) /

o

I.:!.."I = 2(1 + 1) _ -+

o

! Lei gicii sai lftm khi bien d6i bieu thuc

... 1 I • X x).' z . x x

V \ sm2 + cos 2 == sm 2 + cos 2

') I

= NT + 2 - ~ +, ; {T, = r· Cu~ d~,3", • rt.: ((: t-I

, , , 't{ ~1/11 ,TlI'..t("i\ /,~1 ~ , /1 ('. .

t.3. PHAN TICH CA.C NGUYEN NHAl\i DAi'1 TOI SAL LAN! CU,\

. ..: .... 1 , ,

HOC SINH PHO THONG TRUNG HOC KHI GV\I TOAN

..

Chung toi chu yeu nghicn cuu nhung nguyen nhan ve kien thuc ella HS da dan toi sai lam, Doi voi nhung nguyen nhan ve ~ thuc va y chi cua HS, nhLlng nguyen nhan ',Ie nang luc chuyen men va.pharn chat nghe nghiep cua _GY, cluing toi se ban den khi neu ra cac yeu cau can thiet de thuc hien co hieu qua cac bien phap duoc d~ xuat 6 chuong 2.

1.3.1. Nguy~ n nhan 1: Bieu khong day dzi va chl nlt xdc cdc thuiic tinb ella cdc kluil niem to all hoc.

Chung ta biet ding: Kluii niemla mot trong cdc san phdm cua tu: duy todn hoc, ~loi khai niern deu co 1l9i ham va ngoai dien, T~p hop cac

_ ... _,_ -_ .. -._-----

dau hieu dac trung cho ban chat cua cac ct6i Wong duoc phan anh

trong khai niern chinh 1a noi ham cua khai niern. Tap hop cac doi Wong

,. "

().)

co chua cac dau hieu tren chinh la l!gO~lI dien cua khai niern, Viec khiing ndm vieng nlji luim va ngoai dien cua mot khdi niem se dim :1S toi SIr hieu khiing tron ven, thtim chi sai l~ch ban chat ctui khai niem. Ttl do, cac sai l'am khi giai roan se xuat hien. Mat khac nhieu khai niern trong roan DQc la rno rong hoac thu hep cua mot khainiern co truce do. Viec

- I

HS kJuJllg ndm vicng khdi niem nay se di dan tci viec khong hieu va khong

the' co bieu tu ong ve khai niem khac. Nloi quan h~ giua cac khai niern

. '" "

trong, roan hocco tinh lien ket logic. Nhieu khai niern kho trong roan

hoc moi duoc dua VaG trong,ch'trong trinh PTTH nhu : vecto, bien hinh, nguyen ham, rich phan ... Neu chung ta kh6ng kip thoi co nhung co gang hoan thien rnoi v~ phuong phap giang d9-Y cac khai niem thi HS se rat kho khan trong viec linh hoi cac khai niern do ...

Nhieu khi nguoi ta hay noi tai 51:! "mat gee" cua HS ve kien thuc IJ; 1 truce her can hieu ding: do 13. Sl{ "mat gac" }Ie cac khai niem. Khorig hic§'u su me r¢ng khai niern gee hinh hoc sang khai niern gee luong giac thi HS g~p ngay kh6 khan trong vice narn vLing khai niern ve cac ham .ILlQng giac va t11 d6 viec bieu dien g6c IUQng giac, viec gia: cac ph uong trinh, dac biet vi~c'giai cac bat phuong tr inh luong giac se khon~. tranh khoi sai lam. Nhie u HS da viet sinx ~ 1 => x < ;r/2 + 2lv-r, hay khi giai h¢ phuong trinh IUQng giac thi cac so nguyen khac nhau deu duoc ky hieu la k va d~n toi su thu hep t?P nghiem. Ngay hai don vi do gee IUQng giac la dQ va radian rna HS ciing khong hieu duoc day Ia

.hai don vi do khac nhau dan toi each viet sai lam 60° + Zkzr (?). Mot .sai l'am kha ph6 bien Ia nhieu HS kh6ng biet ding: khi viet y = sinx va y' = cosx th i x phai do boi don vi radian, do do viet sai rang y' = 0 ~ x = 90° + k.180° (?).

64

(;'

HS khong hieu khai niern anh X? se dc1n rai khong narn vling cac khai niem v;' phep bien hinh, ham 56. ham 56 hop. ham 56 pg~Qc. HS khong hi{~!J khai niem ham hop. nen nhan dc;tng cac ham hop con yell, tv do HS tinh sai dao ham ella cac ham sO' hop, ch~ng han y :: sin (_ 2x) thi y' == cos (- 2x) (?). HS kh6ng hieu kha: niern ham nguoc, nen tuong ding hai ham sO' y == x2 va y :: {X nguoc nhau, nhung lai kh6 hieu khi nghc n6i ham so y == 2x Ia ham so nguoc ella ham so y == Iogzx (?). HS khong hieu khai niern ham so don di~u nen nhan djnh sai Ihrn "ham so y == I/x Iuon nghich bien" rna le ra phai n6i "ham so nghich bien tren hai khoang (- 00 ; 0) va (0 ; + 00)".

HS kh6ng narn duoc khai nierr; gioi han cua day so se d~n rOi mot loat 5V khong hieu cac khai niern tiep theo nhu : gio: han ham so, tinh lien tuc ella ham 56, dao ham, nguyen ham. tiern can cua cac duong congo Tham chi nhieu HS con hieu cac kY hieu 00, + 00, - 00 la cac so, nen san sang viet: 00 - 00 == 0, ° . 00 == 0, 1'" = 1 (?). Mot so HS con hieu "x tien toi a" tire la "thay x boi a" ("). M¢t so thi nghi dng "hai duong tiem can nhau thi kh6ng c~t nhau" ( ?). Nhieu HS kh6ng hinh dung n6i kha; niern tiep xuc ella hai duong nen khang dinh sai Iftm la "tiep tuyen tai diem uon cua duong bac ba khOng tiep xuc voi dUong bac ba do" (?) (chi vi thay tiep tuyen dac bier nay "di xuyen qua d6 thi"),

HS kh6ng hieu khai ni~m ve can thue nen da viej_jaT == a hm' {;.? == - a, tu do dc1n rei sai Ihm khi giai phuong trinh va khi bien d6i

.~. ~""--'.-~'"'' - - - •.... '~."' .. '- ,.--~---.--.--~-~-"-,,---,.~'"-'.-"- .. --.- '- ".-~ - --~ ------.----.---,------._--_- ---._--_

cac bieu thuc. HS kh6ng hieu khai niern cue tri cua ham so (eve dai, cue tieu) , nen khong ph an biet duoc khai niern nay voi khai niern ve gia tri Ion nhat va nho nhat cua ham so" dac biet la su lam dl:lng kY hieu

65

66

HS kh6ng hieu khai niern nguyen ham, d~n toi viec chung minh h~ thuc giua c.C:c nguyen ham bang each chung minh "dao ham hai ve bang nhau" (?). Le ra phai hieu rang nguyen ham clla m¢t ham so f(x) 13 m¢t t~p hop cae ham F(x) sao eho F'(x) = f(x) nen chung minh' hai

- ,

nguyen ham bhng nhau, tuc la phai thea nguyen tac chuna mi .

tap hop bang nhau. ~~) ,0 /, ~W.1ttl:~;_1

HS khong narn viing khai niern vi; M true toa d¢"bi; cac v~g .f~ goc, nen nhieu khi lay dun vi do tren hai true toa d9 khdc nhau de "dl C ve" do thi ella mst ham so nao do.

HS kh6ng narn vilng khai niern ve parabol nen da nham I~n khi

, ,

goi ten mot so du._~_?$ c6 ~inh dq.ng hoi giong parabol, ch~ng hq.n duong

y = x '. HS kh6ng narn duoc khai niern guy tich nen nhieu khi moi lam

xorig phan thuan da dieridat sui: "guy rich cac diem thea man tinh chat cua bai toa,n la duong.'." Tharn chi luon nghi guy rich phai la mot duong, chu kh6ng baa gio la mot mien mat phang toa do, dac bier kh6ng baa gio guy tich chi gorn 1 diem (I),

HS co khi con nhiim Ian khdi niem vtri dinli Ij, chang han vi kh6ng narn duoc khai niern so rmi 0 cua luy thua nen da "chung minh 2° = 1".

HS khong nam duoc mci quan he gnla khai niern s6 rnu thuc va khai

I

niem can thuc nen cu tuang 2~ = xa voi rnoi x thuoc R, tu d6 dan toi

1

giili sai phuong trinh x{2 = 2 vi dua phuong trinh ve dang 2~ = 2 roi

dan toi nghiem x = 1 (?).

HS khong hieu khai niem v1; nghiern cua mot h¢ phuong trinh nem ~ nhieu khi ket luan h~ hai an x, y e6 "ha_i nghiern la x = 1 va y = 2". r ~~w Nhu vay, qua cac d~n chung cu the tren chung ta c6 the thay nr viec kh6ng narn vilng cac thuoc tinh cua khai niern, HS co the' bi d~n

roi cac sai lam trong loi giai. Chung toi xin luu y toi nguyen nhan nay vi neu GV kh6ng co cac bien phap su pham k~t) thai thi chinh tit d6 se gay ra hau qua Ion cho HS. the hien qua so d6 sau (so d6 1) :

I Kh6ng phat hi$n J [ Kh6ng pIlan nch I

)-"1 I-raia_'o vien, l

l Kh6ng cling co I ~ r

I [ Kh6ng phan IO?i I

1.3.2 Nguyen nhan 2. : Khong ruim vu-ng cau trUc lOgic coo djnh lb' Dinh Iy Iii mot rnenh d~ da duoc kh~ng dinh dung. C3:u true thong thuang ella dinh 19 co d;mg A "" B. Trong cau true ella dinh 11' ,A ". B thi A lil gia thie] cua dinh Iy va cho chung ta biet pharn vi su dl:lng duoc

, ('

dinh ly, Nguo, ta con n6i A la di~u kien du de c6 B. Nhung kha nhieu

I Nhan dang sai I
Khong rulm vung
nQi ham
I I T
Bien doi sai
f1

E
Kh6ng ruim vti"ng I Ki hieu sai ]
cac thuoc tfnh H
Khai niern I J I
Chirng minh sal E

N
I Ve hlnh sai ]
S
Kh6ng nam virng I Dien dat sai I ,,\:
ngoai dien r
H9c sinh

....

khi giai toano

HS lai khong narn vung hoac coi thuon'g gia thiet A nen d~n tei sai Iftm

67

Khi hoc dinh li Viete thuan, nhieu HS chi nho t6ng va rich hai oghi~m Ia bao nhieu, chu khong de 9 toi gia thiet Clla dinh 19 la phuong

- "'---_-----------

tdoh phai I~ phl1ong_ trinb._Q_~c hai c6 nghierrrja ;c O~ L\ ~ 0). Tu

do, HS se mac sai l'am khi ap dung dinh li nay.

Khi hoc dinh Iy ve d~u cua tam thuc bac hai, HS chi nho la neu

i

n. < 0 thi f (x) =ax2 + bx + c luon cung dau voi h~ so a rna khong hru 9

a ;t: 0 (xem thi du 1 muc 1.2.5 trang 30).

'-Jhi'f.u HS nharn <!ifl thie't A cua dinh Iv cunz la dieu kien c~n de

> '. '_' •

c6 ket luan B nen mac sai lam. Khi hoc dinh 19 v'e chieubien thien cua .. ham so "Neu f'(x) > 0 voi moi x thuoc (a.b) thi ham so y = f (x) d6ng

, • J

bien tren (a;b)", kha nhieu HS nghi day la dieu kien c~n va du de ham so

y = f(x) dong bien tren (a.b). Thuc ra day chi ia dieu kien du (ham so

y = xJ la ham so dong bien tren R, ham so y = {X d6ng bien tren [O:~)

nhung khong thea man gia thiet cua dinh 19), Tu d6, HS Sl1 dung dinh ly'

nay de xac dinh tham so sao cho ham so d6ng bien tren (a;b), dc1n toi

thieu cac gi'a tri cua tham s6 thea man yeu cau bai roan (xem thi du 4,

rnuc 1.2.8 trang 46). Khi hoc dinh ly : "Ne~l f'(xo) = 0 va f"(x,) > 0 (f"(xJ < 0) thi ham s6 dat cue tieu (cue dai) tai x = xo", HS cung m3c ~ ~~~m khi g~p tlnh hu6ng f(xo) = 0 va-f"(xo) '= 0 lai ket lu?n ham~ ',,,,,,,~ ... so khong co cl,1c d~i va cl,1C tieu (!). Tinh hucng nay chi dc1n den mot

suy nghi hop lyla tra ve guy rac 1 xet Cl,1C tri ham so nho dao ham bac

nhat, ~an thi dl,1 cho _:~cua HS la ham y = x-l: Khi hoc dinh 1y Veiersrtrass ve su ton tai gioi han day, nhieu HS cung tuang di~u kien

~ cion di~u 1a di~u. ki~n cfm va ty luan sai l'am "Day khong don dieu

_, .. ------~-.- .. -----

...

nen khong c6 gi6i han" (").

68

Khong ndm viing gill thiet cua dinh. Ij nen HS co the tip dung

.

dinh Ij ra ngoai phlJm vi ella gill thiet. Chang han, hoc guy tac tinh

,

, dao ham cua ham s6 y =vx", HS kh6ng luu y ding s6 mil phai la hang so nen da ap dung guy tAc tren de tinh dao ham cua ham s6 y = x', N gay. HS

PITH rna con n6i ding ... x' + '1 1 la so nguyen khi va chi khi x + 1 chia

X" + x +

het (?) cho x2 + x + 1 mac du x thuoc tap s6 thuc R! Dieu tren chi n6i duoc neu x + 1 va x2 + x + 1 nhan gia tri thuoc t~p cac s6 nguyen Z. Khi hoc v~ bat dAng thuc Cauchy, HS kh6ng d€ y tdi gia thiet chi ap dung bat dang thuc cho cac so kh6ng am nen khi gap bai toan so sanh x + I/x voi so 2 da ap dung ngay d€ c6 ket luan sai lam x + l/x > 2 voi

x ;0 I va x + 1/x = 2 voi x = 1. p. Le fa HS xet x +.1 = I x I + .1 2: 2

. , : \ / AlL /. n c-Tt~ ~ .. !(~ dV~~ lL~('il

de di toi ket lu~n dung. ) ;!~ ~ ~ - Ri~JlLUnlt.J

,. _:z." ,

Nhieu HS lop 12 van dung dinh Ii Newton - Leibnitz de tinh tich

, ,

l . . . .

Phfm!, 7 m~c dO. ham s6 !<hong xac d~ _"~_lie.~_~ct~: ._.~.:~~Qc!-.:; 1]

de c6 dap so sai la -1,5, thuc ra tich phan nay kh6ng t6nl tai. Dinh !9 ve cac phep roan cua gio! han day chi phat bieu cho gioi han' cua mot tong huu harrday va cac day nay phai c6 gioi han, nhung nhieu hoc sinh ap dung cho tong vo han, tharn chi kh6ng can tung day cho truce phai c6 gioi han (xem thi du 1, muc 1.2.7 trang 39). Nguy~nbhan nay con d~n toi viec hoc sinh su dung sai cac "a hang ruing thuc" khi giai roan, chang han

~ ab = {a ~w, 10&( xy) = lo&-'{ + log.y.log, b . 10gb C :k loga c ...

....

'{ \

T6m lai, viec kh6ng ndm viing cau tnic [~gic cua dinh Iy se dc1n

HS tdi nhieu sai 1:1~ tr~~g ~h~_!l~'~ ~;;;_~~~giai ~~~~_~~-Ch·~~~··t6i-xin luu

, l ...

Y boi SO do sau (so d6 2)

69

_I DlNH LY : A => B I
l J
.
I Khong narn virng A J- Khong nam vUng B

1 J I
Khong Khong Sfr dung Sfr dung Co A
co A co A dinh If B rna Co B nhung
van suyra wang tu 1- khong suyra suyra
suy ra khOng chua dung nhaA co A khong
B co B phai B

I J
I I
LOI GIAI SAl


- -,
- [Hoc SINH I

,I GIAO VIEN I

1.3.3. Nguyen nhan 3 : Thlsu cae kiln thtic din thilt lie lOgic.

Suy luan la mot hoat dong tri tue dac biet cua ph an dean - mot rrong cac hinh thuc cua tu duy. Hoar dong suy luan khi giai roan dua tren co 56 ella logic hoc. HS thieu cac kien thuc c:an thiet ve logic se

"--- .. _---------------

mac sai Cam trong suy lu~n va tu d6 dan den cac sai lam khi giai toano

. .. - .. _ .... - .... _.

Truce het, nhieu HS chua narn vung cac phep roan cua dai so

rnenh d~ : phil dinh, keo theo, tuyen, hoi, tuong duong. Ngay viec su

... -._.---_._-_ .. _._-- .. _---_

dung tLI noi "va". "hoac" vc1n la dieu kh6 khan cua 'nit nhieu HS. Le ra

.. _: ..... ':"::'_-_._ •.. _._--. . ~ - .. - ' --_ .. __ . __ .. _ ... .

...

can khang dinh : "tam giac can hoac vuong" thi lai khang dinh "tam giac la vuong can". Khi bien d6i phuong trinh tich AB = 0, HS van viet A = 0 va B = O.

Vi~c khong c6 y thuc ve phep tuy§n va phe~E¢i~ay __ cE_~IiS ·kh6. khan ngay eel viec linh hQi cac ~~~LI!i§,!DL._<;.::l~inh_lY:_l':!hi~y __ d.iI?:111Y_~6

-.-- .. '.' ... -.-.--- .. -----.- .. -------.-.-----~-------~

gia thiet va ket luan mang cau true tuyen hoac hoi. Nhieu tinh chat dac

-----_. -------- .. _---------_._---- -'--.--.-- ... - .. -.--~---,.---.

trung ella mQt khai ni~m eling c6 cac kieu celu truc nay. Cht1ng han,

70

1_'- \,t'\A.A. v )

djnh 19 "N eu ham sO' dar cuctri qi :} = x' thi ham so khong co dao ham tai x = x' hoac dao ham tai do trier tieu''. Nhieu H: khong hieu duoc tv do suy ra mot khang dinh " Neu ham sO' co d90 ham tai di~m cue tri x = x' thi dao ham tai do phai bang 0".

I

Khong nam vung rnoi quan h~ gnia phep phu djnh va cac luong

tv 'ii, 3, HS rat de phat bieu sai cac menh de, nhan dang sai cac khai niern va nhieu khi dtin toi cac phep chung minh sal. De khang dinh ham sO' y = 3x + 1 khong phai la ham sO' chan, nhieu HS chung minh f(x) ;e f(-x) voi Vx. rna khong biet rii~g:.p~~ dinh m~nh de "dung voi mQi x" la menh de "sai vai it oha! mQt gia tri <;ua x". Nhu V?y chi dm neu f(l) ;e f(-I) la

-

xong phep chung minh. De chung minh phuong trinh co nghiern, HS luon

nghi roi cae dieu ki~n co nghi~m cua.phuong trinh (rna nhieu khi khcng c6), It de y vi~c chi ra m¢t nghi~m c~ ·the la xong. am chi, khi GV chi ra mot nghiern r6 rang rna HS van chua chiu cho day la phep chung minh (CU tuc)ng phep chung minh phai la mot 19 luangi d6 that ghe gornl).

. ,

Khong narn dUQC phep phu dinh, HS rat kh6 khan khi phat

.-- •. - ._--,-------------"._ -e- .... _ ..... __ .. _--

bieu menh de phan dao cua mot dinh 19 de suy ra menh de pharr

-- t'

d a 0 d 6 c tl n g I a' m Q t din h 19. T u d] n h 19 "N e uta m t h u c b <) c h a i f(x) = ax:! + bx +c co- ~ s 0 thi af (x) ~ 0 voi 'ilx:', neu HS l~p duoc menh de phan dao thi co ngay dinh 19 "N eu 3~ sao cho af(a) < 0 thi f(x) = ax' + bx + c co hai nghiern phan bier", De chungminh ham so y = f(x) kh6ng lien t1:1C tai x = x' , hoc sinh nhieu khi khong riLIng 19i sau khi chi ra su vi pham mot dieu kien rna i?i tiep tuc tim each chi ra S1) vi pharn ca nhung dieu kien khac va dan toi be tac trong lei giai vi nhieu

khi ham so do chi vi pham mot dieu kien rna thoi (cha .. ng. han ihil!~_st

sinx , .' If' /1 ( {f' (1 v

. { -X-,VOI X ~ 0 Ltw UU(',~ .. '£1.'/

f(x) - 0 voi x = 0 L"a~(n -r~ , i' ,

71

thoa man

Lnhung lim f(x) ~ f(O) nen ham 56

:1:-0

khong lien tuc tai x .' 0). Khong B<lm duoe phep phu. djnh, HS rat kh6 khan khi dung phuong phap chung minh phein chung. Viec "phu dinh

-- ... _-----

khong hcan toan" se d~n toi sai Jam trong lai giai phu dinh a > 0 la

a < 0 gay cho loi giai thieu truong hop a =, 0 (xem thi du 6, muc 1.2.3

trang 25). . IUd ~ lul-tt." ~ Ct~ ~

: Phep toan keo thea ~uallogic la phep toan rat q an trong trong

--.---~----- .. --- .. -" .. ,-_._- ,:._. __ .,_._- ---

viec phat bieu cac dinh 19, khai niern va trong lap luan cua lei giai. f

:Tr~ng ·l~i2."~hU;; toi ca phan tich nguyen nhan HS khong narn viing { cau true logic ella dinh 19 nen d~n toi nhisu sai Iil.m khi giai.toan. :'hung1h,1 su 'thie'u hieu bier v~ logic, mil dac biet Iii ~~oan keo_the?)I~i Iii ilc, "nguyen nhan cua nguyen nhan" d~n den cac sai lam. Nhieu HS khongt hieu dau la dieu kien can, dieu kien du va tharn chi the nao ia dieu·lv.

--." --~- ,._---- .. _-__ .---~

ki~n din. ~he nao la dieu kien du, HS nhieu khi cling kho tra lei. Viec L(

'SLI \--il,1ng cac ttl "neu'', "thi'', "vi", "do do", "rna", "nen", "boi vay", "dan

d.~n": "tv", "suv rail chuaduoc HS su dung dung._!i~~~_!uong trong bai giai cua HS tran ng~p ky hi~u =>, ~ mQt cach tuy ti~n khong pheii su dang mung cho vi~c logic h6a lai giai, Nhieu truong hop HS viet A => B nhung A khong phai 1a dieu kien du de co B. Tham chi khi tim dieu ki~i1 can va du, HS van di~n dat mot tu "de" that phi logic: "De phuong trinh ... co nghi~m khi va chi khi ... ", le ra phai bo ttl nay. MQt "sang kien" cho viec khong dung cac phep keo thea hoac tUdng duong khi bien d6i cac menh de la HS cu viet xong rnoi menh de lai xuong dong rna giila hai dong khong he co kY hieu logic gi ca ! Biet bao nhieu HS thuong ban khoan : bai quy rich nay co can lam phan dao hay khong ?

HS con thilu nhii ng hiell biet ve cdc quy tdc suy luiin nen dan toi nhieu sai [am khi thuc hien cdc phep ehli-ng minh, Pharr tich cac suy luan

'":'?

1-

73

trong chung minh to.in hQC, ta thfiy mo~ chung minh bao gem mot so buoc ClJ ban, rna moi buoc duoc thuc hien thea nhung quy tac nhat dinh goi la cac quy tcic suy luan,

HS nhieu khi nhftm phep suy nguQc tie'n la m¢t phep chung rninh.

Ch~ng h<;ln, de chung minh v6i mQi a, b, c ta co bat d~ng thuc

3 (aZ +b2 +c2) ?: (a +b +C)2 co HS da vie't i l, , J~/';'

3(a2 + b2 + c2 ) > (a + b + C)2 -AJ),~' 1'" ~)~ ,,\, J ..

' i ; , .. ~ ,'/ 1\. ,,!,1z,(

=> 2aZ + 2b~ + 2c2- 2ab - 2bc - 2ac ?: 0 I _',.,-\ \ -/

• j .:. ,1~

i ,j ,/-- I

=> (a - b)2 + (b - ~)2 ~ (c - a)2 ~ O,db bat clAng thuc cuci cu~g hip~, {..II'

nhien dung nen bat dang thuc can chung minh cling dung. ./ t,/,

HS de bi nharn khi su dung cac quy de sai :

A~ B, B A

A ~ E, A 8

Chang han nhiing suy luan sai lam nhu "Nell day so tien va bi chan

tren thi co gi6i han, rna Jay co gioi han nen day phai tien va b! chan tren' hoac "Neu day so tien va bi chan tren thi co gioi han, nhung d;1~, so khong tien va cung kh6ng bi chan trer: __ nen day khcng cogioi han".

HS con chua.hieu thuc chat cua phep quy n~p roan hoc, nhieu

'. - l

khi dung phep tuong tu thay phep chung-rnlnh-b~ng- quy n<;lp tQ(ir~ hoc,

Chang han, de tinh dao ham bac n cua ham so y = e2.'<, HS Ian IUQt tinh y' = 2e2x, y" = 4e2X, y'" = 8e2:< va tuong tv suy fa yen) = 2ne2.'( rna khong chung minh gi the~ (?). Khi chung minh quy nap toan hoc, HSj luon xet n =- 1, mac dii ~ai roan yeu cau chung minh voi n > 2. HS [chua, hieu duoc vi sao lai gia su bai toan dung voi n = k de chung minh bai to an dung voi n = k + l,th~rn chi con thac mac : voi n - k ta chua chung minh thi sao lai dung, duoc:

Ad;f313 A ,<

B. ~ * B tiv }]- ~ :A

A ~ B \f:1 IW,~' ~

B ~ A. 'J '" c t~~,')

• J dr-:(

MQt trong nhiing nguyen nhan lam cho HS dinh huong sai lam ~

khi giai toan, tharn chi kh6ng tim ra loi giai la co nhiing menh d~ B do nhieu rnenh d~ co the dQC lap suy ra dUQC rna HS kh6ng nho duoc her.

v~ 3 hinh thuc suy luan phan chung thuong dung,

- -

A ~ B, B

A

Khi chung minh phein chuE.g'_E_h~ ~~_"~6i 6 .!!.~n _l!.S khong biet phil dinh. m9t m~nh de. Nhieu em tuy biet If! .~ 'iai dAn toi mau thuan thi moi ket thuc phep chung minh phan chung, nhung thuong chi nghi dan den mau thuan voi gia thiet rna kh6ng biet r~ng co khi phai phoi hop voi gia thiet d~ dan den mau thuan voi mot chan ly nao d6 da dUQC khang dinh hoac dan toi hai menh d~" mau thuan nhau. HS khong biet d~y du

,"

Do d6 phep phan tich di len de tim loi giai khong phai hie nao cung thuc hien duoc trong hoan canh HS kh6ng nam duoc cac dieu kien du de co B.

Trang SGK thi cac phep chung minh duoc trinh bay theo phuong phap tong hop rna khong qua phuong phap phantich de dan toi each chung minh, trong khi do thi GV lai khong the hien dUdi dang tuong minh cac kien thuc v~ cac guy luat, guy tac, phuong phap suy luan d5 duoc su dung.

1.3.4. Nguyen nhan 4 : Hoc sinh khiing ndm visng phtro ng phap

...

-giai cdc bai todn co ban

HS khong narn vting phuong phap giai cua cac bai roan eo" ban thi dall toi sai lam trong lei giai,

74

. ('j l.,.

!"_ 'I ~ .

,'I ,~ .:',: { .I l or i', ,I ,I

1', \ ",

r 1('(

, . ,

Kh6ng narn vling phuong phap gi?i, HS khcng nghi duoc du cac kha nang can xet va dc1n tai dat dieu kien sai (xern cac thi du 1, 2, 4,

'1 ') - "0 ..... ,,)

rnuc .,,--.J trang c -.:).:.. .

. Kh~ng.n_~IIl~gDg__phu.ong phap giai, HS se bien luan kh6ng du

. ._ ... ,-",_-,,-,- - .. -- ... --.-~.- .'.'-_-.,._ . - •. _ .....

cac truong hop xay ra cua bai roan .

. - .- ---_.-

Kh6ng nam viing phuo'~g'-phKp £i~-i,HS se ap dung kh6ng dung pharn vi va dc1n toi be tac kh6ng di tai loi giai. Chang han, sau khi giai

cac phuong trinh cu the 3X + 4" = 5", 6:< + 3:< = 2~ HS co the duoc hinh

th.a',~h mot p~r~on.I,g phap giai cho lap cac phuong trinh t + b' =: CX vai, .. __ _.

f' l -.! ; ,~ v.. _i- A _ 5-- ('. I I ' (. I

ii"., \, ,' ... ',',t-! , , --i abc> 0 ,..p~- J/ .:. tv ,r.r .... l

I, .c >,> , ••• -t 0 ' (

'<'Ii. i i)L.:~) ['" 6 -\\~;."tl,,': .' .• i

\, )".' ~:i .-\. \_;_-{I'" a, b > e 'J

. \ ' .. ,), "

\ \.\,: .'- . a b < e

\ I. I'}' ib---' v'

~-,\)'~,.,,:pphuOng trinh co mot nghiern da biet Xo nhu sau: :\ ... Ill,: j

I .-: ," \ I t ... r\. I

J:'-/'" Dua phuong trinh ve dang (~'.) x + (b). x _ 1 \_\1 ... ,_'",:. "-

c ' \ C ' f"

-i j, \,v.

- Chi faX= : x.) lil nghiern • \, \.,'F;

'T" a b 1 hi '( ( a, x (b\:< 00'" bie A' ) la

- i~eu-. - > t It,X) = (-\ + \ -I ng ien nen x =x

C C I.- \ C /, C) o

nghiern duy nhat.

e

b

< 1 thi f(x) nghich bien va nghiern x = < "llng e

a - Neu 0 <

duy nhat.

V<.Ji phuong phap giai nay. HS se thanh e6ng khi giai phuong rrinh

"

..

12:< + 5:< = 13:< (x = 2), nhung se that bai neu ap dung voi phuong trinh 4)( + 6:< = 9:<, Phuonz trinh nay khcnz duns duoc phuong phap ziai tren

~ . ~ :;:,. ~ :;:,

vi kh6ng nham ra nghiern xC). Mac du voi phuong phap giai tren ta van chung

rninh duoc tinh duy nhat cua nghiern. HS quen mat phuong phapgiai

75

76

cho lop cac phuong trinh dang

A(a2)!tX) + B( ab )f(x) + C(b2)f(X) = ()

(b a) f(x)

(_d day A = B = 1; C = -1; a = 2, b = 3, f(x) = x) la dat an phu t =

de dua phuong trinh ve dang AtZ + Bt + C = O.

Khong nam viing phuong phap giai HS co the bo qua nhung buoc quan trong va di ngay tOi ket luan (xem thi du 5 va 1, muc 1.2.4 trang 26- 29).

( 0:

; Khong narn viing cac phuong phap giai qla cung 1 loai toan, HS

l' , .»

khong chon ra duoc phuong phap giai toi uu cho mot bai toan cu the.

1 "

2

Chang han : '11m m sao cho gia tri Ion nha; cua ham so y = x m tren

x + m

[0; 1] dung bang 2", kha nhieu HS su dung phuong phap giai dung dao

ham. chia cac truong hop va I~p bang bien thien cua y tren [0;1], nr do tim m. Loi giai nay van ra ket qua dung nhung 'qua phuc tap. Trong khi d6 ch'i can de y f(O) = -1 voi m :;:: 0 thi HS d~ dang xet hai truong hop m ;c 0; m = 0 va chung minh duoc ngay khong co m thoa man bai toano

Sl,I phuc tap cua phuong phap giai cung voi rihiing nguyen nhan ve tam li fat d~ dua HS toi lei giai sai lam.

Khong narn viing phuong phap giai, loi giai ella HS se kh6ng .co trinh tu logic va se khcng biet khi nao ket thiic lei giai .

...

khang djnh :

Tat ca ket qua nghien CUll 6 chuang nay, cho phep chung toi

1\ r c

i ~ \.l v

:': HS con mac nhieu sai lam khi giai toano

I), -\jj{'"

, \

. \

\1

1 \._

I l! .

Ll \. .c- ', \-

• \ \1..

,!. Nhiinz sai Cam cua HS co the he thong lai de giup GV de phat

- .;

hie n trong loi giai cua HS.

'" Nhung sai lam khi giai roan cua [-IS xu at phat tv nhieu nguyen

., ' . i - \:\ I .' /"\

, '~k'''' h' I I· i'o j ''1'"

nhan ve 'len t uc. ! ".Jv\~IJ: . ,./III.,!.I"II_ : 'i" ', '.,.'.j

_/ ~ v /

* Ttl nhilng nghien cuu nay, chung toi co co so thuc tien va ll luan de d~ nghi cac bien phap hieu qua nharn phan rich, SUa chua va han che cac sai lam cua HS khi giai toano Ttl d6, g6p phan hoan thien Ii luan day hoc men roan va ren luyen nang h:1C giai toan cho

HS PTTH.

'"17 I I

Chiro ng 2

C4~C BIEN PHAP REN LUYEN Ni\NG i.uc GL~I TOAN CHO HOC SINH PHO THONG TRUNG HOC THONG QUA PHAN 'ncn vA SUA CHUA SAl LAlYI

2.1. CO so Lf LuAN

"

Trang qua trinh nghien cuu d~ xay dung cac bien phap han che' va sua chua cac sai lam cua HS khi giai roan, chung toi da dua tren co 50 li luan khoa hoc sau day.

2.1.1. Li luan ve phtrong phap day hoc

Can cu nay la tien de de bo sung vao phuong phap day hoc men roan nharn dar rnuc dich d'e ra la han che, sua chua cac sa! lam cua HS khi giai roan. Theo Nguyen NgQc Qu.ang, "phuong p'hap dc;tv hQc la each thuc lam viec .;;ua thfty va _eua tra trong_s~ phoi ~.2E__thQ!}g_Ilh~!_~~~_Q_t!6j _

_ .-----

su chi dao eua thay. nhAm lam cho tra tugiac. rich cUC. tu luc dat toi

:r'\IC dich d?y hQc,,'[76, tr. 23J.. KfJ ~----.

Phuong phap day phai co hai chuc nang l~ trL~en dat va chi. dao, Phuong phap hoc cung c6 hai chuc nang la tiep thu va tu chi dao [76, tr. 24].

Phuong phap khoa hoc roan hQC va phuong phap day hoc roan hoc la ditng cau, nhung kh6ng dong nhat. Nguoi HS chi chu d¢ng sang tao trong khuon kh6 cua S\I chi dao su pharn cua GY. cua chuong trinh dao tao, phat hien lai chan If moi cho ban than [76, tr. ~ 1].

Cac nha tam li hoc.khang dinh ding "moi tre em binh thuong kh6ng eo' be nh t~t gi del-l co klui ruing dat du o c hoc viin loan hoc

78

phd thong, co ban. dau cho chuong trinh roan da hien 'dai hoa" [40, tr. 49J.

Nhu v~y co the thay rang cdc sai [am cua HS khi gill; todn la co the khdc phuc duoc.

Giao due hoc man toan lien he khang khit voi mot so khoa hoc khac : khoa h9C duy v~t bien chung va duy vat lich SU, roan hoc, giao due hoc, tam li hoc logic hoc, dieu khien hQC va 19 thuyet thong tin.

Cac bien phap sua chua sai l'am cho HS khi giai roan cling phai dua tren miii lien h~ hiru co cua cdc bq mon khoa hoc tren.

Cac bienphap sua chua sai Cam cho HS. cling nhu phuong phap day hoc noi chung phai phan anh dUQC : cau true ben ngoai va cau true ben trong, d~c bier doi voi cau true ben trong phai chi ra duoc cac thao

' (

tac tri tue, each thuc t6 chuc logic cua su nhan thuc va linh h¢i cua

HS.

...

DOi voi viec chi ra cac sai lam cua HS khi giai roan cung co nhieu quan diem khac nhau tren the gioi,

Nua thecki sau ~eua the ki XIX, mot s6 nha giao due hoc nguoi -) Due rna tieu bieu lit Aphogut Lai eho ding :, viec chu ,'1 toi cae sai lam ( cua HS trong gio hoc co anb hu o ng xdu den viec tiep thu bai giang .. D~c biet q uan diem nay d~ nghi khiing viet lai loi giai sai len bang

r

vi dieu nay lam cung co them sai lam trong 9 thuc HS. (105, tr. 70].

Day la mot quan niern co tinh chat may moe gino dieu, khong dua tr~n qui luat tiep thu tri thuc mot each co 9 thuc cua HS.

.. '. t I

Chung toi d6ng nhat quan diem voi R.A.Axanop : "Viec tiep thu

. i

tri thuc mot each co y thuc duoc kich thich boi viec tu HS phan tich

, J' I .

rnot each co suy nghi m)i dung cua tung sai tam rna HS pham phai, giai

_/

79

thich nguon goc cua cac sai lam nay va tu duy, li luan ve ban chat cua

C' ~,

cac sai lam" [105. .tr. 70].

[ r

'Chinh A.A. Sroliar cung da dat ra mot so bai roan phuong phap

giang day rna trong do lien -quan toi cdc tinb huiing HS mdc sai tam khi

,il, I ,

gilli todn va da khang dinh 'Can phai co bien phap nham day hoc mon

I,

roan dua tren cac sai lam, khi cac sai lam cua HS xuat hien [106,

tr. 89-93].

Mat khac, ngoai cac phuong phap day hoc truyen thong, cac nha nghien cuu ve phuong phap day hoc da dua ra mot so phuong phap rndi rna tinn huiing mdc sai Lam cua HS tao di'eu ki~n de phdt huy TIU diem cua cac phuong phap nay.

Chung toi xin minh hoa ro y tren bang quan diem Cl:1 the duoi day. Phu o ng phap day hoc giai quyet van tie dua tren tinn huiing co van ae trong day hoc. Khi HS mac sai lam 0 loi giai la xu at hien tinh huang d) vrln de, khong phai do GV de ra thea y minh rna tuno nay sinh tillogic! b~n trong cua viec giai toan. Sai lam cua HS tao fa mau thuan va rnau thuan nay chinh la dong luc thuc dgy qua trinh nhan thuc cua HS. Sai lam cua HS lam nay sinhnhu cau cho nr duy rna "tu duy sang tao luon bat dau bang mot tinh huong goi van de" (Rubinstein [53, tr. 115]).

- Sai l'am cua HS xuat hien thi se kheu goi duoc heat dong hoc tap rna HS se duoc huang dich, goi dong co de ti111 ra sai lam va di toi IOi gia: dung. TIm ra cai sai cua chinh minh hay cua ban minh d~u Ifl str khdm phd. Tu su kharn pha nay, HS chiern linh duoc kien thuc mot each tron ven hon. Tuy nhien din gay niem tin cho HS la ban than rninh c6 the tim ra duoc sai lam trong mot lo'i gia: nao do. HS co the tu SUY nghihoac trao d6i d~ tim ra cac sai Iftm.

80

81

Trong tinh trang phan cue trinh d¢ cua HS nhu hien nay (ngay 'trong mot lop) thi phuong phap day hoc phan hoa c6 rae dlfng rut bot d:€m s1:1 phan cue.

GV c6 the doi xu ca bi~t ngay trong nhiing pha day hoc .d6ng loat nharn han che va SUa chua cac sai l~m cua HS khi giai toano

Su phan hoa trong nho thong qua nhiing muc d¢ "bay" sai I~m khac nhau cho tung doi tUQng HS, the hien ngay 6 viec GV giao bai

' ,

tap tren lop hoac bai tap ve nha,

Su phan hoa ngoai nho thong qua cac cong viec t6 chuc hoc t?P thea nhorn, t6 va phu dao rieng cho nhilng HS mac nhieu sai l~m tram

trong. . I Le~ ",[)4)1 ) Jet; ((,uI '

. Tuy nhie~. chiing ta kh6,ng d:Qe q~en tan ~~n1 e~~ . 'nYcu11

cae phuong phap day hoc truyellUlong vao rnuc dle~.Ehf'g I! I

huong toi. :

Cac bieriphapduoc de xuat deu dua tren quan diem hoar dong

'1 ,

~rong phuong phap day hoc voi cac tu tucng chu dc;loda duoc G.S

Nguyen Ba Kim due ket nhu sau :

a) Cho HS thuc hien va tap luyen nhling heat d¢ng va hoar d¢ng thanh phan tuong thich voi noi dung va muc dich day hoc.

b) Gay ct¢ng co heat cong va tien hanh heat d¢ng.

c) Truyen thu tri thuc, dac biet la tri thuc phuong phap, nhu phuong tien va ket qua cua heat dong.

d) Phan bac heat d¢ng lam ch6 dua choviec dieu khien qua trinh 'dC;ly hoc [53, tr. 73].

2.1.2. Nhu'ng van de co- ban cu a tam If hoc day hoc .

Ngay trong cac nguyen nhan dan toi sai l~m cua HS khi giai roan

' ,

, ,

c6 cac nguyen nhan ve tam ll cua HS. Chinh vi vay khi dua fa cac bien

"

a) Tri thuc va nhung ki nang, ki xao tuong ung voi tri thuc ay [a ctoi tuong cua heat dong hoc, Viec linh hoi tri thuc, ki nang, ki xao ella xa hoi se khong th~ thuc hien duoc neu nguoi hoc la khdch. the bi ~ng ella nhii ng lac dQng SIC pham.

b) HOC;lt dong hoc lam cho chinh chu the hoar d¢ng nay thay d6i va phat trien, Chi co thong qua do nguoi hoc moi gianh duoc nhung kha nang khach quan de ngay cang tu' hoan thien chinb minh,

c) Hoat dQng hQC can lam cho HS c6 ca nhung tri thuc ve heat dong hQC rna chung ta thuong goi la phuong phap hoc t~p [31, tr. 62-64].

Cac bien phap sua chiia sai 111m cho HS khi giai toan phai ttic

dqflg va nham dich vao hoat dqng hoc cua HS. .Truoc het can tao ra dong co hoc tap sua chua cac sai I§.m. HS. phai thay viec sua chua cac sai lam khi giai roan 1a mot nhu cau va can phai tham gia nhir mqt chu the mot each t~( nguyen, say me hao hung. HS phai co duoc "dong C(J hotin

phap su pharn, chung toi lay cac qui II:1*t cua tam ll hoc day hoc lam co so Ii luan.

Nho Ii thuyet heat dong rna nganh tam li hoc su pham da nghien cuu duoc nhieu ket qua do thuc tien day va hQC d~t ra.

Chung tci rattan thanh quan diem "suy cho cling giao due la qua trinh bien nang luc ella loai nguoi thanh nang Iuc ella m6i tre em" [31, tf. 59]. De lam duoc cong viec nay can phai thong qua hoat dqng df:ly hoc, Tat nhien, hoat dqng day khiing the tach riri hoat dqng hoc,

"Chat luong heat dong hoc phu thuoc VaG trinh dQ dieu khien va t6 chuc (trinh dQ nghe nghiep) ella thay, ket tinh 6 trinh dQ phat trien nhilng hanh dong hoc t?P tich cue cua HS" [31, tr. 61].

Chung toi rat quan tam toi ban chat ella heat dong hoc cua HS da duoc khans dinh :

. ~.

82

...

X3

thien tri thuc". Can lay hoar dong hoc t~p ella HS de lam co so cho qua trinh Iinh hoi tri thuc.

HOC:lt dong hoc cua HS phai thong qua cac hanh d¢ng cu the: hanh dong phan tich, hanh dong cu the hoa,

Can cu vao nhting ket qua nghien cuu ve tam Ii hoc day hoc, chiing toi thay can hinh thanh 0 HS nhung nang lire t{UJ ra nang luc, rna trong do ban than nang luc tim ra cac sai lam khi giai toan se tao ra nang luc giai toan cho HS. Tu do HS tt;r tin di sua chua cdc sai lam.

Tam li hoc khang dinh "muon hinh thanh khai niern 0 HS phai lay hanh d9ng cua cac em lam co SOli [31, tr. 107]. Neu t6 chuc hanh dqng cho HS kh6ng tot thi HS khong the nam vilng cac thuoc tinh cua khai niern va nguyen nhangay ra sai lam se xuat hien.

, Hen mia, cac bien phap phai tap trung vao phat trien heat dong, ren luyen cac ky nang hoc tap cua HS (ky nang nhan thuc, ky nang thuc hanh. 1--.1 nang t6 chuc hoar dong, 1--.1 nang tu kiern tra, danh gift).

.• .~~.__/

2.2 BA .mUONG CHAlv~ CHi DA~V .DV~G CI\~ ,BIEN PHAP SU PHANI NHA.NI HAN CHE VA SUA CHUAi CAC

-, < I., ., ~

SAL LANI--CUA HOC SINH KHI GIAl TOAN.

2.2.1 Phuong cham 1 : Tinh kip thai r

Cac bien phap phai chu y thich ung voi thai diem thich hop. Bien

"

phap chi phat huy hieu qua neu duoc ap dung dung hie. Khong t~e tiiy tien trong viec pharr tich va sua chua, cung nhu han che cac sai 15m cua HS. Dac bier, tI~gilt'ii"IIiaGV tiep xiic true tiep voi HS IA e6 han. SI,1 kh6ng kip thai se gay lang phi thoi gian va GV se kh6 c6 dieu kien

-.., r

!

lay lai tho! gian da mat.

Tinh kip that cua cac bien phap doi hoi su nhanh nhay cua G V truce cac rinh huang dien hinh, nharn tac dong dung heat dong hoc

cua HS. Tinh kip thoi doi hoi Sl,1 rich Cl,1C hoa hoat dong nhan thuc cua

1 '

cit GV va HS.

Tinh kip thai doi hoi GV phai nghien cuu va du dean duoc cacl

sai l'am cua HS 0 tung thai diem cua nam hoc, tung gio len lop. I

Tinh kip thai doi hoi G'V Iuon 0 tu the thuong tr\lc voi muc tieu dey PQc nharn han che va SUa chda sai lam cua HS khi giai roan, Sai lam cang sua muon bao nhieu thi su vat va cua "thay va tro cang tang bay phieu. Tinh kip thoi doi hoi GV phai viing vang v~ tam li nghe nghiep, biet chu dong trong thai do, biet kim che khi kh6 chiu va biet

f i

d6ng cam voi moi di~u sai, dung cua HS.

J

i Tinh kip, thai doi hoi GV phai tranh thu giao tiep vci HS, khong

chi 6 tren lop rna con trong nhieu hoan canh khac de t~n dung co hoi thuc, hien cac bien phap day hoc,

, Tinh kip thoi doi hoi GV phai tim each han che cac nguyen nhan jl

sai 111m cua HS ke cakhi cae sai la~_ehua xuat I]jffi, -Pi~ / ,II

i Tinh kip thai con doi hoi GV phai cung co thuong xuyen cac sai

Cam da sua chua cho HS, nharn khong de cac sai lam tai dien. ,

R ( -r..

- I ~"?J" CjV--lVl1 =r

2.2.2 Phurmg cham 2 : Tinh chinn xdc. . O«} \~ I o..~"

s« chinh xac trong loi giai la doi hoi cua tcan hoc, cling la Sl,ltoi

hoi cua nhiern VI! day hoc man roan trang nha truong pho thong de .rdao tao co chat IUQng nlulng nguoilao dongmo.".

Cac bien phap d~ xuat phai di toi muc tieu lam cho loi giai cua HS bao dam dQ chinh xac cao.

Tinh chinh xac doi hoi GV _ph~i d~!!_c!~_~hinh._ __ xac, ttl ngon ng11 J thong thuang den ngon __ ~~~~~~_~_!l_Qc._GV phai la mau muc ve phuong ~l phap, tu duy chinh xac, ve lei giai chinh xac cho cac bai toano d

II ~ Tinh chinh xac doi hoi GV phai chi ra chinh xac nguyen nhan sai \Ji

J " I \'

lam cua HS trong loi giai. Jji C£6J .l\) dl~ Xw 1 ~ ~

84 ~?~\_J«l~ - ~1V1

\ ~,f ~I:U ~ ~ ~ c1/J

85

r ("" 1\,r- r ) _J" l )](' hmj Jtu: I~

I), l.{) th) 'Vt-y - J .--

''0 ' \/p I ,II

, \_ ~'

lt~~ tL J dV1'

GV khong duoc phu dinh loi giai sai .eua HS mot each chung chung. H Tinh chinh xac doi hoi cac bai roan ella GV dua ra khong duoc

, sai lftm. Doi voi HS gioi thi e6 the su sai lftm ella bai roan se duoc HS phat hien, nhung doi voi HS yeu hoac trung binh thi bai toan sai d~ gay hoang mang va mat niern tin vao GV

Tinh chinh xac doi hoi su danh gia chinh xac muc dQ sai l~m ella HS. Chang han, khi HS viet 2lC.2Y = 2lC.Y thi thong thuong, cac GV cho day III mot sai Iftm nghiern trong v~ kien thuc co ban. Tuy nhien, d6i voi mot so HS cu the thi sai lam nay e6 khi chi do su vo y thuc gay nen.

Tinh chinh xac doi hoi GV danh gia bai giai cua HS qua diem s6

I

mot each cong bang,

Tinh chinh xac doi hoi GV phai biet huang dftn dieu chinh, sua

1

ch iia mot loi giai sai de HS tu tim ra mot loi giai dung. Tinh chinh xac .doi hoi GV phai lua chon dung bien phap t6Ci uu 1

trong tung tinh huang dien hinh.

1.2.3 Phuong cham 3 : Tina gitio due.

,-

Tinh giao due dOl hoi GV phai lay su phat trien nhan each cua

HS lam muc tieu cho' cac bien phap,

Tinh giao due giup cho HS thay duoc tarn quan trong lcud su chinh ~ac trong loi giai,

Tinh giao due giup cho HS tranh duoc cac sai lam khi sai Iftm

ch ua xu at hien.

Tinh giao due giup cho HS xac djnh duoc dong co hoc t~pi man toan, Tinh giao due doi hoi GV phai co pham chat va nang luc xung dang la nguoi thay.

Tinh giao due doi hoi GV khong lam cho HS bi xiic pham ve nhan each khi macsai lftm trong loi giai,

Tinh giao due lam cho HS co. 9 chi trong hoc roan, giai toano HS khong ngai kho, biet kien tri va ccln .nan d€ di tdi loi giai dung. Tlnh giao due giup cho HS' co nhling thoi quen tot, nhu biet t1,1 kiern tra viec lam cua rninh, biet phu dinh sai lAm cua chinh minh va biet giup ban nhan ra sai lfim.

Tinh giao due giiip cho HS khong d~u d6t, dam hoi khi khong hieu, khong biet, tranh gian l~n quay c6p de mong loi giai dung.

Tinh giao due giup cho HS tich cue suy nghi, tang cuong hoat dong hoc dua den su ham me chiern linh kien thuc chuan xac.

Tinh giao due doi hoi GY phai biet khen ngoi, khich I~HS khi da sua chua duoc sai lam.

Tinh giao due lam cho HS thay duoc moi sai lam deu co the sua chua duoc neu tim ra nguyen nhan va c6 y chi khac phuc,

Tinh giao due lam cho HS biet duoc uu diem cua true giac la co the giup nghi ra, kiern tra loi giai nhung cung chinh true giac co the dun HS den cac sai lam.

Tinh giao due doi hoi GY dam nhan ra sai Cam cua minh trong loi giai, trong each danh gia HS.

Tinh giao due doi hoi GY khong nong vQi trong viec thuc hien cac bien phap de mong muon cham dut ngay sai lAm cua HS. CO" nhiing sai lam doi hoi GY phai huy dong nhien bien phap d6ng bo va qua mot thoi gian dai rnoi khac phuc noi,

Tinh giao due doi hoi cac bien phap phai dua tren tinh thuang yeu hoc sinh, mong HS tien b9 va tuyet doi kh6ng xiic pham hay guy ket sai nguyen nhan sai lam cua HS.

Ba phuong cham tren ho rro, be> sung cho nhau lam cho cac bien phap thuc hien dung muc. dich va ke't qua. Tinh kip thoi lam cho tinh giao due dat duoc nhanh han va nguoc lai tinh giao due giup cho cac bien phap thuc hien duoc kip thoi, thuan loi han.

86

Tinh chinh xac cung co cho sinh giao due va tao di~u kien cho tinh kip thai. NguQC lai, tinh kip thoi la chuan bi di~u kien the hien tinh chinh xac,

Mot bien phap, mot hoat dong cua GV hay HS nhieu hie the hien ca ba phuong cham chi dao quan trong tren, Chang han SI:! tich cue h6a trong viec nhan thuc cac khai n iern vua co' tinh kip thai de phong cac sai Iftm, vua e6 tinh chinh xac de dat dUQC su hieu biet sau s~c khai niern va co tinh giao due trong viec giup HS ch"U

dQogehie'm Hoh cac kien thuc chuan. ~ f dl.t r

2.3

I )i

2.3.1. Bien phap 1 : Trang hi ooy dli, chinn xdc cdc kiln thiic ve b~

miin Totin.

"

Bieri phap nay gia! quyet bon tinh huong cu the sau day:

* Tinn huo.ng 1 : Dq,y khdi niem todn hoc nhtr the nao de trdnn sai tam cho HS khi giai toan ?

Ngoai cac heat d¢ng day hockhai niern da duoc O.S Nguyen Ba

I

Kim va P.G.S vii Duong Thuy trinh bay 6 (53J, chung toimuon nhan

manh va hoan thien themm¢t so bien phap.

G v can d;;' dodn trtroe (bang kinh nghiern ban than hoac trao d6i voi d6ng nghiep), cac kha nang khong hieu het nhiing thuoc tinh cua khai niern. Chang han doi v6i khtii nism ham so nguoc thi HS co cac kha nang nao kh6ng hieu het thuoc tinh ella khai niern ? Du xay dung qua khai niern song anh di chang mia, cudi cling doi voi HS pho thong, chung ta d~u dtra va~ phtrongtrinn f(x) = y voiy thu~c t4p, giti tri ctia ham/pho tnroc. Neu phuong trinh nay co nghiem duy nhdt thl chung ta co the xay dtrng dtrac ham g sao cho neu f(x) == y thi g(y) = x va ham g goi la ham nguoc cua ham f. Kh6ng nhung the, nguoi

87

88

ta cia th~ thu hep t~p xac djnh cua f d~ t6n tai g. Nhieu HS khi noi

~ , .

toi ham f ·huang chi quan. tam toi viec f eho boi I bi~u thuc giai tich,

rna khong de j toi nhisng ham f cho boi nhteu 'su« thu» gilli tich, tham chi eho boi cac each khac nhu bang gia tri tuong ling, tflj thi v.v ... Nhieu HS khong de j tai t~p xdc dinh cua f. Chang han ham y = f(x) = x2

I ._.

la kh6ng don dieu tren R, nhung don di¢u tren R+: HS coi ham y = f(x) = x!

tren R va tren R+ la nhu: nhau vi cung mQt ctich tuong ling mdi x vqi y = x'.

, Tu do, HS hoar dong nhan dang va the hien d~ mac sai I~m. M¢t so HS con noi ham y = arcsinx ta ham nguoc cua ham y = sinx, chit

kho~g nhan rnanh 111 ham nguoc ella ham y = sinx voi x E [ - ~ ; ~]. HS con nghi rang hai ham nguoc nhau f va g ia khdc nhau !

1 x+l

Do d6 khi tim ham nguoc cua ham y = x hay y = -; y. =--

x x-I

HS ngo ngang kh6ng dam ket luan ham nguoc cua ham f chinh la ham f.

Khi khong co phuong trinb f(x) = y thi HS khong biet tim ham nguoc nhu the nao. Chang han cho ham so cho boi bang gia tri tuong ung (bang 7) :

x 1 2 3 4- 5 I 6 I 7 I 8
I
I-~' .16._ .... 15 13'--' ---. ··--t ._-- .. - '--1
! 8 7 4 I 2 I
v 1 I
. ..

.. ".... ' ... ~~

thi HS khong tim ra ham nguo c.

Co khi HS con kh6ng nam duoc neu ham g la ham nguoc cua ham f thi ham f cling Ia ham nguoc cua ham: g.

Neu du dean duoc cac sai Iftm tren thi chac chan GV se chuan bi bai giang cua minh de de phong truce sai lam cho HS. SI:l' chit d9ng tie phong sai lam xudt hi~n bao gio cung mang tinh tich cue hon la 10 sua chua sau nay. Nhiing sai l~m cua HS v~ khai niern roan hoc mang dau an kh6 phai va nit mat cong chinh lai cho chinh xac,

o day cung luu y phan bier viec chua hidu het voi hidu sai. C6 nhiing khai niem kho, HS khong hi~u het cac thuoc tinh ngay m9t luc rna phai qua cac hoat dong nhan dang va the hien moi di toi su tron ven. Chinh viec chua hieu het cac thuoctinh cua khai niem se rat de

.. . -e- .

d~n den viec hidu sai khai niern. Do d6 c6 nhiing sai lftm cua HS phai lam cho HS hieu het cdc thuoc tinh cua khai niem thi moi mong HS het hieu sai. Chang han ngay viec dung ki hieu y rrmx' Y min til lop 10, lop 11 duoc HS coi la ki hieu cho gia tri Ion nhat, nho nhat cua ham so, nhung luc d6 HS chua hieu het duoc cai sai rna phai len lop 12 thi HS moi

duoc giai thich thoa dang.

Trong li thuyet thong tin, de chong nhieu thong tin, nguoi ta d~ ra cac bien phap rna chiing ta c6 the van dung van phuong phap day h9C [76, t~p 1, tr. 65-81].

HS co nhiern vu giai mii thong tin rna GV dua de'n. Lam sao de

. : :1

viea stsc giai rna. cua HS ? Hay ph~i doi mil de' HS d~ giai1 mii hem. Cac

bien phap vt! giao CI:! true quan da duoc nhieu tac gia nghien cuu, 6 day chung toi de cap toi mi)t bien phdp doi mil cho HS bang each nang gid mang cua thong tin. Chang han, khi day khai niern ham so lien tuc, :

chung ta co dinh nghia :

..

"Ham so y = f(x) goi Ia lien tuc tai diem x = Xo neu :

1) Xo la mot diem thuoc tap xac dinh cua ham so, ;

2) lim f(x) = f(xJ" [11, tr. 115].

Mot dinh nghia khac :

"Mot ham so f(x) xac dinh tren tap so D, goi la lien tuc tai diem x E 0, neu lim. f(x) = f(xo)" [59, tr. 150].

Thuc ra khi viet f(xo) dii mang thong tin La Xo thutc t~p xdc dinh, nhung cac dinh nghia tren deu nha~ manh rieng yeu caunay chinh la tao di~u kieri cho HS giai rna tot hon (de hen !).

89

Tham chi, thea chung tai la ra~ tot, trong [59] cac tac gia con luu y ngay duoi dinh nghia :

"Nhu v~y mot ham sO' f(x) lien tuc tai diem Xo neu va chi neu ba dieu kien sau duoc thoa man dong thai:

1) f(x) xac dinh tai x = xo'

2) lim f(x) ton tai, .

3) lim f(x) = f(x )".

Chung ta thay voi luu y nay cac tac gia tiep tuc tip) dtiu ki#n gilLi nui cho HS. Hay hinh dung Ie ra chi eLin 3, nhung dii them 1, va cuoi cung them clL 2. Hieu duoc dung j su pham va jnghia thOng tin nay, GV se t6 chuc heat dong nhan dang t6t han.

Trong heat dong nhan dang thi cac phan thi du gil! vai tro nit quan trong trong viec tranh cac sai lam cua HS khi linh hQi khai niem. NQi ham mang cau true hoi thi chung ta dua ra Ian luot cdc phim thi

du vi pham tung thuiic tinh clia khdi niem. Chanz han, ham so y = x + 2

". .... . x + 1

{X2 - 1

voi x ;c -1

khang. lien tuc tai x = -1 (vi pham 1), y = X+o 1

voi x = -1

{I,. 0

kh ~ l'" . '1 (vi h 3) - - VOl X:;C khc I'"

ong len tuc tat x = - VI P am ,Y = x ong len

1 voi x = 0

tuc tai x = 0 (vi pharn 2).

Khi HS da di qua mot loat khai niern, GV can chi ra miii lien h~ giica cdc khdi niem bang cac SO dO rat d~ ghi dau an cho HS. Chang han, rnoi lien h~ giila 3 khai niem quan trong cua giai tich (so d6 3) :

...

f(x) kha vi tren (a ; b)

f(x) lien t\IC tren (a ; b)

f(x) kha tich tren (a ; b)

Ngay viec phan loai khai niern, GV cling can chi ra s~r piuu trdn theo con duirng todn hoc da di de HS thay duoc noi ham va ngoai dien

Day so

C6 the. GV con phai biet "btic trann loan canh" ve cdc khd! niem quan trong trong mot phan rnon Toan, chang han cac khai niern quan trong rrong chuong trinh Dai so - Giai tich 6 PITH (so d6 4) :

----------------------j T~phQP

I

I

,

'---II Anh X? I

Ham so

; I

I, Tinh 'd~n di~u I

I TInh don di~u I

Cac day so d?c bi~t

.t'

I Gi6i h~n ham so

Gioi h~n d~y sO'

Vi p~an I

Nguyen ham

I Tich phfm I

-{ Phuong trinh ;I-~ --. I

! Belt <ting thue 1--1 Ba:t phu~ng trinh 1-'

1

. .

Bien d6i tuong duong

91

;i

b a

if

:;: Tinb huo ng 2 : D~y cdc dinb Ii todn h(JC nha the nao de HS trdnn 1- sa; lO.m khi giai todn ?

N6i toi dinh li toan hoc la noi tei m9t khiing dinh. dung (du chung J tel c6 day phep chung minh dinh li hay khong). Tuy nhien, viec quan trsmg rna GV can quan tdm dliu tien fa cdu true LOgic clia dinh li, Nhu chiing toi da phan tich 6 1.3.2, viec khong nam vilng cau true dinh 11

,

se d~n HS toi sai lam khi giai toano Cac dinh li toan hoc thuong duoc dien dat thee cau true A => B. Ai c~ng biet A 1a gia thiet va B la khang

~. dinh, ket luan cua dinh li, Nhung chung toi xin luu y them: A cho biet dung dinn Ii khi nao va B cho bier se ket luiin, suy ra dtro c gi khi co A.

D<;IY dinh Ii toan hoc co the dUQC thuc hien theo hai con d}\onfc : con duong suy dien va con duong co khau suy doan [53, tr. 188]. ~~c:.

Nharn han che' va de phong cac sai : I~m cua HS khi gia: toanO ~ chung toi thay can thiet phai phiin tich ro gia thiet cua dinh Ii. HS .nhieu

Ll khi kh6ng quan tam toi gia thiet dinh li rna chi quan tam tOi ket Iuan cua djnh li nen dftn toi sai I~rn.

GV e~n nhan manh gia thiet cua dinh If c6 cdu true h9i hay tuyen.

Chdng han, dinh Ii Viete :

Neu phuong trinh bac hai ax' + bx + c = 0 (a ;: 0) co nghiern xl' :s "thf t6ng va tich cac nghiern cua no la :

Cau true cua gia thiet c6 cautnic h¢i : fa ;: O} 1\ {~ ~ O}. Truoc khi dung dinh li nay phai kiern tra hoac dar dieu kien de bai toan thoa mao dong thai hai dieu kien cua gia thiet. HS rat hay quen dieu kien

a ;e O. Nhieu HS vc1n tinh tong va tich cac nghiern cua phuong trinh ":.

x2 - X + 1 = 0 mac du phuong trinh nay vo nghiern.

92

GV can tao ra nhilng thi du rna cac dieu kien cua gia thiet chu a thoa man hoan toan de HS thay ding moi dieu kien cua gia th. ~'t Ia khong the thie u dtroe.

Thi du khi day dinh Ii : "Neu ham so y = f(x) co dao ham rex) = 0 [ten khoang (a ; b) thi Y la mot ham h~ng so tren khoang d6, nrc Ia f(x) = c voi moi x E (a ; b) [35, tr. 54].

Nhieu HS chi d~ y toi viec f'(x) = 0 rna khong d~ y tei (a ; b)

nen dan tei sai Ifun nhu 0 thi du 3, muc 1.2.8 (trang 45). .)

GV c6 the su d1,lng thi du nay de luu Y : dinh li phat bieu cho mot khoang (a ; b) chu khong phat bieu cho hop eua nhieu khoang, Neu gap hop eua nhiiu k~oang thi phai van dung dinh n cho tung khoang thi ) moi tranh duoc sai lam.. -, . /

Khi dinh li co cau true A :::::> B thi A la dieu kien du de c6 B chu chira chile id dtiu ki~n can.

GY cungcan neu ra thi du de thuyet phuc, chu kh6ng chi dung I~i 0 Vi~c. nhac nho ..• cac.,t ... hi., ~\l rna dac b~.' ~t I~ .~~i'hd1Jl'~'~' ~~-;baot::3iu

cung. ttpJ an tUf!ng1sau dOl VOl HS. t- ti'1.\, eta (;; thJ ~

- ~ ",

, Chi1ng h~n,khi day dinh Ii :' -- ->

l'

"Cho ham s6 y = f(x) co dao ham rex) tren khoang (a ; b). Neu

.) (

f'(x) > ° vdi rnoi .x E (a ; b), thi Y = I'(x) d6ng bien (nrc fa thuc su

tang) tren khoang d6" [35, tr. 54).

-,

GV can chi ra ham y = x3 thuc su tang tren R nhung y': = 3x2

van trier tieu tai x = 0, tharn chi ham y = (X thuc su tang tren [;0 : ~)

nhunz v' = ')~ kh6n2: xac dinh tai x = 0. dieu nay chUng to gia thiet

'-' • - ~x ..., '. . . ...... ~

...

f'(x) > 0 voi moi x E (a ; b) chi fa dteu ki~n dzl chu: khong phai iid dtell kien can de tranh sai l~m nhu 6 thi du 4, muc 1.2.8 trang 46.

De khac sau dinh Ii, GV can chi ra dinh li nay la khai, quat cho dinh li nao rna HS da biet truce d6. Chang han dinh Ii ham so cosin la khai quat cua djnh If Pitago. Dinh li ham so sin lam HS nhih I~i dinh li v~ quy tich cung- chua g6c.

9~



94

...

Khi day mot djnh 11 can chj ra cho HS cac huung ling dl:lng czia dinh. Ii de t90 ra S1:l" nhay cam ezia HS khi dung truce mot bili toan biet nghi toi viec van d\lng dinh Ii nao,

Chang han doi voi dinh li Lagrange 0 chuang trinh giai rich lop 12, giao vien c~n chi fa 3 huang ung dl:lng cua dinh li nay: chung minh h~ thuc hoac nit gon bieu thuc, ehung minh phuong trinh co nghiem, chung minh belt dang thuc.

Chung toi xin d~n ra 3 thi d\l cho J.huong, flng' d\lng nay. Huang 1 : thi du 3, muc 1.2.8 trang 45.

Huang 2 : "Cho rn > 0 va a, b, c la cac s6 thoa man

a + b + ~ == O.

m+2 m+l m

Chung minh : phuong trinh ax' + bx + c = 0 co nghi~m thuoc (0 ; 1). Rai roan nay HS lop 10 giai nho dinh Ii dao v~ dau cua tam thuc b~c hai, HS lop 11 dua vao dinh li ve gia tri trung gian cua ham so lien tuc, ca hai lei giai d~u kha phuc tap, HS lop 12 co the dua vao dinh II Lagrange.

Qua bai roan nay, G V ciln guip HS thay duoe mot bai hoc thu V! : khi duoc cung dip them cdc kiln thuc q lop tren thi ding co them nhii ng cacb nhin mol ve cung mqt bdi todn va tll do co them nhung lui gitii moi. HS thay ding S1:l" phdt trien ella kiln thu« ehinh La mo- rqng tam nhin, chu khong phai la mang them mot ganh nang trong tri oc.

Huang 3 : "Chung minh r~ng neu 0 < b < a rhi

a - b 1 a a ._ b"

a < nb < b .

Co the goi y HS vi~t bat effing thuc can chung minh ve dang "gan

vai ket luan" ella djnh Ii ~agrange : ~ iii 1f..! ~

, 1 Ina - lnb 1 "'J 0 \_ (;

- < <

a a - b b '

Tv do HS thay ngay : xet ham so y = lnx kha vi tren (0 ; +00)

" ..... . (b h .,( . f(a) - feb) 1 lna- Inb M

n en to n tar cEO; a) sao c 0 f c) = a _ b <:>;;- - a _ b . I it

III

o < b < c < a nen b > c > a' tv eac ket qua tren suy. ra di~u phai

chung minh. .

Di~u dac bier can luu y Ia khi day dinh li toan hQC cho HS la : G V can cho HS thay r6 phu ong phtip phdn tich de chung minh djllh fi.! Chinh bien phap nay giup cho HS de di toi chung minh dung trong giai roan sau nay. Day dinh li chinh nharn muc dich truyen thu noung tri thirc phuong phap lien quan tOi phep chung minh.

* Tlnh huang 3 : Cung ed.a cae kien thu» VI logic nha thl ndo de HS trank sai Idm khi gilli tcdn ?'

NhAm giai quyet tinh huang nay, chuong trinh man to an PTTH chuyen ban da chinh thuc dua "mot so hinh thuc suy lu~n toan hoc" v~lO phao man D~i so lop 10. GV khi day theo chuong trinh cai each giao due din tharn khao them van d~ nay d~ tien hanh bien' phap cung cap cac kien thuc -ve logic cho HS ngay tu d~u cap PTTH. [

Truce het 'cffn luu y tdi rnenh de va cac phep roan rnenh de nhu

h' di h h ~ . ' ~. k ' h d (", / .~ I -t '-h" (w' /IA-i

P u inn, 9L tuyen, eo teo, tuong uong. tel" If MLI,' "

Theo thuc nghiern cua chung toi, vi~c dua ra cac thi tiu thea ngo.n ngil tl,1 nhien can di truoc cac thi d_~Jlleo ngon ngu toan hQC.: Day chinh la con duong di tv "true quan sinh d9ng" den "nr duy truu tuong" cua nhan thuc, Chang han, co tht! neu menh d~ A = {Troi nang} ; B == {DQi mu} thi thong thuong hoc sinh ducc nh~c'nh6 "Neu troi nang thi d9i rrni" nen HS de hinh dung ra y nghia cua phep keo thea A => B.r,

A la du d~c6 B, nhung luu Y la nhieu khiHS van d9i rrul khi troi . khong nang, nghia la A chua phai dieu kien can d€ co B:

Dac bier, neu A => B la dung thi day Ia mot thi du d€ nhan manh

rnenh de dao B => A kh6ng dung. HS co the thdy ngay viec minh d¢i / mu khong lam cho troi niing !

95

. 96

i'

M9t thi du khac, de phu dinh menh de do mot ban neu ra : 'To ;thuong xuyen tap the due buoi sang~1 thi chi can chi "a mot bu6i sang rna ban ay khong t~p. Tu do d<1n den m6i quan he gitla hai luong tu "voi moi" va "ton tai", Tu ngon ngu thong chuang, GV bc"it dilu su dung cac khai niem, tinh chat, dinh 11 to an h9C rna HS da biet de phan tich tinh chan li ella cac menh d~ Ii, hoi, tuyen, phu dinh, keo theo, tuong dUdng cua cac menh d~ cho truoc (cac khai niem, tinh chat, dinh li roan mQc co the lay (, chudng trinh PITH Cd so thi HS cang de hinh dung).

Chang han, neu A= {sO' tu nhien co t~n cung la O}; B = {so tu nhien co tan cung la 5}; C = {so tu nhien chia het cho 5} thi ta co (A V B) :::. C dong thai C :::. (A v B), do do (A v B) ¢> C Iei tieu ~hu~n ehia he't cho 5 ella so tu nhien. Khi kiem tra mot so chia Mt cho 5 haykh6ng chi can kiern tra A hoac B. Tu do phu dinh menh d~ nay ta

-

co (A i\ B) ¢> C, qua day HS thay duoc them rnoi quan h~ giua cac

phep roan hoi, tuyen, phu dinh va tUdng dUdng.

. i Mot sai I~m can tranh Ia 11 dl:ingeaC1Q1u··-~-u-1-6-gi:-e-t-u-y-t-:· ... -~ Nhieu

HS viet A :::. B rna A kh6ng phai dieu kien de co B. ChAng han, co HS viet "x 2: 1 ~ .x = I", Ie ra viet fIX 2: 1 va x s· 1 ~ x = I". IG hieu keo thea (~) doi khi con duoc viet trong cau van : '~Doan xe di :::. rn¢t dam

~ ... f~"tV~~~ y~' .~J~~(){cf.

. Viec trang bi cac kien thuc logic khong chi dung lai o "mot so ~ hinh thuc suy luan roan hoc" trong phan dau chuang trinh Dai so IOP..eJ~

to rna can duoc thuang xuyen cung cO'. '

Phep chung minh quy n~p toan hQC, co nhieu dip de GV khac ro Ch0 HS. Ch;1ng han khi chung minh h~ thuc

? _ nCn + 1)

l+_+ ... +n_ 2

thi ngoai each da d40C SGK trinh bay, GV nen yeu cl:lu HS dung them

/ .

each thu hai nho phuong phap quy n~p toan hoc, Khi day HS tinh dao

ham bac cao, GV lai mot lftn mla ~u dung phuong phap nay de giai roan. Can phan bi~t suy doan v6i suy dien. Chang han de tinh dao ham bac n cua ham so y=e~\ HS co the suy dean yen) = 2n. e2.'( nhung phai chung minh, tot nhdt 6 day Ia phuong phap quy nap roan hoc.

GV cftn khong tuong minh day hoc sinh phep tam doan luan de khAng dinh, phu dinh, Iva chon, bAc cau.

GV c6 the chu d¢ng dua ra cac suy hl?n sai de HS phan tich va tranh yap phai sau nay ..

Dac biet, can lam cho HS nam duoc phuong phap phan tich di len, phan tich, tong hop, phan chung, quy nap.

GV can tan dl:lng bat cu co h¢i nao, mien la hop Ii, de khAc sau kien thuc logic cho HS. Chang han 6 lop 10 doi voi h~ phuong trinh f

r bx + y = a;n~: -l..-c~ ~ ,.~ 1 x + by = cl + c 7 ytc kJ,;' ..tw -# .

thi viec phan rich hai yeu cau sau day la khac nhau chinh la tang cuong

kien thuc logic: ') ~

- Tim a sao cho v6i moi b lu6~ ton tai cde M co nghiem J' ~

- Tim a sao cho ton tai c de 'h~ c6 nghi~m voi moi 5. - -L)~

HS nam; vung cac kien thuc ve logic se han che duoc nhieu sai ~ lam khi giai toano

* Tlnh huang 4 : Trang bi phuung phdp giai cdc bai loan co ban nhu the nao de tranb sai lam. eua HS khi giai loan ?

C6 the n6i rang cac Ioai roan co ban trang chuong trinh Dai so - Giai tich PTTH dell co phuong phap giai, Viec trang bi cac phuong phap giai nay chinh lam cho HS co di~u kien nam viing cac loai toan co ban.

'.. falX + b.y = c1

Phuong phap giai he b-

, a.,x + ..,y - c..,

., w.. ~

dUQC de cap trong chuong trinh ph6 .thong hai Ian de cung cap toi 4 each giai : phep the, phep C9i1g dai so, phuong phap do thiva phuong phap dinh thuc,

97

Potrebbero piacerti anche